Practice Exam v3: Block 5

Lakukan tugas rumah & ujian kamu dengan baik sekarang menggunakan Quizwiz!

A 40-year-old Caucasian man presents with a rapidly enlarging skin lesion on his right shoulder. Upon further questioning, he admits to long-term outdoor exposure as required by his occupation as a construction worker. He further notes that the lesion has been changing colors over the last 2 months, first beginning as solid black then changing to light brown and red, but he denies any bleeding or pain to the area. Physical exam confirms a 7 mm macular lesion with irregular borders and a mottled color pattern on the right shoulder. Question What risk factor serves as the best predictor of metastatic potential of this lesion? Lesion location Presence of ulceration Color variegation Breslow thickness Serum LDH levels

Correct answer: Breslow thickness Explanation This patient's presentation is most consistent with malignant melanoma. There are four major types of melanoma; three of these are characterized by a period of superficial, "radial" growth, during which it increases in size but does not penetrate deeply. These three types are superficial spreading melanoma (the most common type of melanoma), lentigo maligna melanoma, and acral lentiginous melanoma. It is during this period that these variants of melanoma are most capable of being cured by surgical excision. The best predictor of metastatic risk is the lesion's Breslow thickness. The Clark level, which defines melanomas on the basis of the layer of skin to which a melanoma has invaded, does not add significant prognostic information and is only used in the staging of thin (T1) melanomas. Other important factors recognized via the staging classification include the presence of ulceration, evidence of nodal involvement, serum lactate dehydrogenase (LDH), and presence and site of distant metastases. Another determinant is anatomic site; favorable sites are the forearm and leg (excluding the feet), and unfavorable sites include the scalp, hands, feet, and mucous membranes.

A 75-year-old African American man presents with a 5-month history of gradually progressive dyspnea that is especially pronounced when climbing stairs. He also has been noticing that his ankles and lower legs have "gotten larger" over roughly the same time period, which prevents him from fitting into his sneakers any longer. He denies fever, chills, chest pain, palpitations, cough, pleurisy, calf pain, abdominal complaints, sick contacts, or travel. His psychosocial history is noteworthy for chronic alcohol use. His physical exam reveals tachycardia, bibasilar rales, jugular vein distention (JVD) of 5 cm, an S3 gallop, and 2+ pitting edema to the level of the mid-calves, bilaterally. Question Ico-delete Highlights What is the expected echocardiogram finding in this patient? Asymmetric septal hypertrophy Diffuse increase of ventricular thickness without ventricular chamber dilation 4-chamber dilatation with thin left ventricular walls Presence of valvular vegetations Abnormal segmental wall motion during systole or diastole

Correct answer: 4-chamber dilatation with thin left ventricular walls Explanation This patient's presentation is significant for dilated cardiomyopathy. Dilated cardiomyopathy occurs more often in the African American than in the Caucasian population, and in men more often than women. Often, no cause can be identified, but chronic alcohol abuse, major catecholamine discharge, myocarditis, being postpartum, and doxorubicin are frequent causes. On echocardiography, 4-chamber dilatation is a common but not uniform finding. Also present is an increased LV end-diastolic diameter and volume with decreased fractional shortening, thinning LV walls, increased E point-septal separation, LA enlargement, and limited mitral and aortic valve opening (caused by low stroke volume). Intracardiac thrombi are frequently observed and are most often found in the LV apex. M-mode imaging of the mitral leaflets may demonstrate a B bump, or notch, just before systolic valve closure, indicating elevated LV diastolic pressure. Mitral annular dilatation and secondary MR are common. Asymmetric septal hypertrophy is found in hypertrophic cardiomyopathy. Typical echocardiographic findings of restrictive cardiomyopathy include a diffuse increase of ventricular thickness in the absence of marked ventricular chamber dilation and marked biatrial enlargement. Valvular vegetations are indicative of infective endocarditis, while abnormal segmental wall motion during systole or diastole signifies ischemic coronary heart disease.

A 32-year-old pregnant woman presents for a screening ultrasound at 16 weeks gestation. The ultrasound notes that the fetus is male, has shortened long bones (all below the 5%), and has a large head circumference. Question Ico-delete Highlights You refer the patient to a tertiary care center for further evaluation and explain that you suspect that the fetus may have what condition? Trisomy 13 Trisomy 18 Turner syndrome Achondroplasia Down syndrome

Correct answer: Achondroplasia Explanation Achondroplasia, in which bony growth of the long bones of the skeleton is abnormal, is the most commonly recognized skeletal dysplasia. Increased head circumference size can also be observed in the skeletal dysplasias. Over 100 disorders resulting in skeletal dysplasia have been reported. Therefore, additional ultrasound imaging and genetic testing are required for a definitive diagnosis. Down syndrome (Trisomy 21) sometimes leads to shortening of the long bones in utero, but head circumference is often decreased. Ultrasound markers would include the presence of congenital heart disease (present in 1/3 of all live born cases), duodenal atresia, and an increased nuchal translucency. Trisomy 13 is often fatal in utero or in the first few days or weeks of life. Global growth retardation (not just long bone shortening), renal anomalies, cleft lip/palate, and severe central nervous system malformations may be noted on ultrasound. Trisomy 18 is often fatal in utero or in the first few days or weeks of life. Overall growth retardation may be present, which would affect the entire skeleton rather the long bones alone. These fetuses also typically have renal anomalies, severe cardiac malformations, and/or omphaloceles. There may also be a shortening of the sternum and prominent calcanei on the feet. Turner syndrome causes few major structural problems in fetal development for those who survive the first trimester, as many will spontaneously miscarry. Ultrasound imaging may identify a short neck, an increased nuchal translucency (possibly a cystic hygroma), and/or webbing of the neck. Turner syndrome is a sex chromosome disorder caused by a missing X chromosome (all or part). The most common abnormality is 45X (40-50%), also known as monosomy X. Gonadal dysgenesis and mosaicism 47XXX may also result in a Turner syndrome phenotype.

A 28-year-old man presents with a rash. The mildly pruritic lesions are located on his arms and legs. They have been present for about 3 days, without change or resolution. He has tried over-the-counter anti-itch creams, but they have been ineffective. The patient reports that he was seen approximately 1 week ago for some blister-like lesions on his penis. He was given an antiviral medicine and those lesions resolved. He is wondering if he was misdiagnosed and if the 2 "rashes" are related. He admits to feeling some malaise over the last 2 weeks, but he is otherwise healthy. He denies fevers, unusual travel, medication use (except for as listed above), and known allergies. He has no known chronic conditions. On physical exam, vitals are normal; the patient is in no apparent distress. A pink-to-red papular rash is observed on the backs of the hands and feet and extensor surfaces of the arms and legs. The individual lesions are quite distinct; they have a red center, and they are surrounded by a pale ring and then another outer ring of red, inflamed tissue. The remainder of his physical exam is normal. Question The patient is at risk of recurrence for both "rashes." What medication would best prevent recurrence of this patient's second rash (the one presented today)? Acyclovir Diphenhydramine Doxycycline Methotrexate Prednisone

Correct answer: Acyclovir Explanation The correct response is acyclovir. This patient is presenting with a target-lesion rash; it is typical of erythema multiforme, which is a relatively common type IV hypersensitivity reaction. In about 50% of the cases of erythema multiforme, no cause is identified. Other causes are infection and medications. When a cause is identified, recent herpes simplex virus (HSV) infection is the most common. In this patient's case, he likely was appropriately diagnosed and treated for his HSV infection, then developed the erythema multiforme rash. Erythema multiforme is an acute, self-limited condition; it can range in severity from a rash, mild malaise, and pruritus (erythema multiforme minor) to a much more severe condition, involving mucosal surfaces and possible desquamation with erythema multiforme major. For individuals who have experienced erythema multiforme in response to HSV, acyclovir is recommended to prevent recurrence. Diphenhydramine is an antihistamine and is often used in the acute treatment of the erythema multiforme, but it is not recommended for health maintenance as a way of preventing recurrence. Doxycycline is a tetracycline antibiotic. It is useful for several bacterial infections and several dermatologic conditions, ranging from acne to anthrax. It does not have a role in the treatment or prevention of erythema multiforme. Methotrexate is an immunosuppressant medication used for a variety of conditions, such as rheumatoid arthritis and severe psoriasis. This patient's rash is not suggestive of psoriasis, so methotrexate would not be appropriate. Prednisone is a steroid medication, often used during the acute stage of more severe erythema multiforme. Due to significant side effects associated with long-term use of systemic steroids, prednisone would not be recommended as a prophylactic treatment for this patient.

Case A 23-year-old woman presents with dysuria and left flank pain. She is 27 weeks pregnant. Physical examination reveals a temperature of 38°C (100.4°F); there is tenderness with percussion over the left costovertebral angle. Urinalysis shows 15-20 WBC/hpf and 15-20 bacteria/hpf. Question Ico-delete Highlights What is the most appropriate management plan? Schedule renal ultrasound. Order urine culture and arrange follow-up in 24 hours. Obtain intravenous pyelogram. Treat with oral trimethoprim/sulfamethoxazole. Admit for intravenous antibiotics.

Correct answer: Admit for intravenous antibiotics. Explanation The patient should be admitted for IV antibiotics. Urinary tract infections in pregnant patients are associated with significant risks greater than in non-pregnant patients. Mechanical pressure on the ureters and bladder by the enlarging uterus, progesterone-mediated relaxation of smooth muscle, incomplete bladder emptying, and increased urinary tract volume contribute to the increased incidence of urinary tract infections during pregnancy as well as the increased severity of associated complications. Approximately 5-7% of pregnant women have asymptomatic bacteriuria. Untreated bacteriuria is associated with a high incidence of prematurity and fetal wastage as well as a 20-40% incidence of acute maternal pyelonephritis. Patients with uncomplicated bacteriuria should be treated with ampicillin, amoxicillin, nitrofurantoin, or a cephalosporin. Pregnant patients with pyelonephritis (fever, chills, and flank tenderness) are at increased risk for sepsis and preterm labor and require hospitalization for aggressive therapy with parenteral antibiotics. Thus, the option to order a urine culture and arrange follow-up in 24 hours is inappropriate for this particular patient. Sulfonamide antimicrobial agents (such as oral trimethoprim/sulfamethoxazole) are not routinely used during pregnancy. They should be avoided entirely during the third trimester, as they increase serum bilirubin levels and the risk of neonatal kernicterus. Imaging modalities such as a renal ultrasound or an intravenous pyelogram (IVP) are more appropriate for evaluating the anatomy of the urinary tract system, including the bladder and kidneys. Imaging is not routinely used to diagnose pyelonephritis. A renal ultrasound is more useful when renal calculi are suspected. An IVP utilizes a contrast medium to evaluate for renal calculi, chronic pyelonephritis, and persistent hematuria. In pregnant women, renal ultrasound is the preferred imaging modality to avoid contrast or radiation exposure.

A 35-year-old Hispanic man presents for an appointment but is too embarrassed to tell the nurse his chief complaint. You enter the room and coerce him to give you the reason he has come in to seek medical treatment. He admits to severe, intense itching around his anus that has been worsening the last several weeks. He further states that he has noticed increasingly severe and tearing pain in the anal area with each bowel movement. He would rank this pain as a 10/10 on a pain scale and it lasts hours afterward. This intense pain makes him not want to have any bowel movements. He admits to only 1 episode of a small amount of bright red blood on the toilet paper as well as on the stool itself. The patient denies fever, diarrhea, or ever being diagnosed with inflammatory bowel disease. Question What is this patient's most likely diagnosis? Perianal abscess Crohn's disease Anal fissure Anal cancer Internal hemorrhoid

Correct answer: Anal fissure Explanation This patient is most likely suffering from an anal fissure. Anal fissures most often affect infants and middle-aged people. The majority of fissures are considered primary and caused by local trauma, such as passage of hard stool, prolonged diarrhea, vaginal delivery, or anal sex. Presentation of anal fissures is a tearing pain accompanying bowel movements, as well as bright red rectal bleeding that is limited to a small amount noted on the toilet paper or surface of the stool. The patient described all of these components. Patients will also complain of perianal pruritus or irritation, which he also admits to experiencing. According to Bleday and Breen, anal fissure should be suspected based on a history of anal pain that is provoked by defecation and lasts for hours afterward, often with associated anal bleeding. The diagnosis can be confirmed on physical exam by either directly visualizing a fissure (usually in thinner patients) or reproducing the patient's presenting complaints (anal pain) by gentle digital palpation of the posterior (or anterior) midline anal verge. Patients who have a perianal abscess will also experience severe pain, but it's not necessarily associated with bowel movements. There is also visible perianal swelling and erythema. There may also be signs of a fever and malaise and, if the abscess has begun to drain, the presence of purulent rectal drainage. Patients with Crohn's disease often have extremely variable signs and symptoms and possibly will have these for many years prior to diagnosis. Fatigue, prolonged diarrhea and abdominal pain, weight loss, and fever with or without gross bleeding are considered hallmarks of Crohn's disease. Signs and symptoms of anal carcinoma include rectal bleeding (occurring in 45% of patients), anorectal pain or sensation of a rectal mass (30%), or no issues (20%). Issues are often confused with hemorrhoids, causing patients may delay evaluation. Over 80% of patients developing anal cancer have detected human papillomavirus. Internal hemorrhoids create principle issues of bleeding, prolapse, and mucoid discharge. They are usually painless unless they are thrombosed (less common); even when this occurs, internal thrombosed hemorrhoids are less painful than external thrombosed hemorrhoids, so 10/10 pain would be unlikely. Bleeding with hemorrhoids may range from bright red blood that is seen as streaks on the toilet paper to actual dripping of blood into the toilet bowl after a bowel movement. Over time, hemorrhoids potentially will develop into a prolapsed state, resulting in a point in time when patients will note a sense of fullness or discomfort along with mucoid drainage that results in further irritation and soiling of underclothes.

A 35-year-old Hispanic man presents due to a sore that will not heal around his rectum and anal area. This lesion has been draining pus consistently for the last week. He has had intermittent pain with this lesion (4/10 on a 0-10 pain scale) that is made slightly worse when he has a bowel movement (ranking it a 6/10). More recently, especially in the last few days, he has noted pain increases with just sitting. The patient also admits to intermittent periods of itching. He denies fever or diarrhea. No past medical history of inflammatory bowel disease. Physical examination of the anorectal area reveals excoriated and inflamed perianal skin with a palpated induration. Question Ico-delete Highlights What is this patient's most likely diagnosis? Anal fistula Crohn's disease Anal fissure Anal cancer Internal hemorrhoid

Correct answer: Anal fistula Explanation The patient in this scenario is experiencing signs and symptoms from an anorectal fistula. Anorectal fistulas are commonly a chronic manifestation of an acute perirectal process that eventually comes from an anorectal abscess. Many fistulas originate from an infected anal crypt gland. Patients with anorectal fistulas usually will present with a "non-healing" area (an anorectal abscess that is draining) or may describe chronic purulent drainage and a pustule-like lesion on the perianal or buttock area. They will only have intermittent pruritus or rectal pain; pain is increased during defecation, sitting, or standing. Physical examination reveals excoriated and inflamed perianal skin. An external opening of the fistula may be visualized. If no opening is seen, palpation will reveal induration just below the skin. Anal fissure is not the most likely diagnosis in this case scenario. Anal fissures most often affect infants as well as middle-aged individuals. The majority of fissures are considered primary and caused by local trauma such as the passage of hard stool, prolonged diarrhea, vaginal delivery, or anal sex. Presentation of anal fissures is a tearing pain accompanying bowel movements as well as bright red rectal bleeding that is limited to a small amount noted on the toilet paper or surface of the stool. The patient described all of these components. Patients will also complain of perianal pruritus or irritation, which he also admits to experiencing. Patients with Crohn's disease often have extremely variable signs and symptoms and possibly will have these for many years prior to diagnosis. Fatigue, prolonged diarrhea and abdominal pain, weight loss, and fever with or without gross bleeding are considered hallmarks of Crohn's disease. Signs and symptoms of anal carcinoma include rectal bleeding (occurring in 45% of patients), anorectal pain or sensation of a rectal mass (30%), or even no issues (20%). Very often issues are confused with hemorrhoids, so patients may delay evaluation. Over 80% of patients developing anal cancer have detected human papillomavirus. Internal hemorrhoids create principle issues of bleeding, prolapse, and mucoid discharge. Bleeding may range from bright red blood that is seen as streaks on the toilet paper to actual dripping of it into the toilet bowl after a bowel movement. Over time, hemorrhoids potentially will develop into a prolapsed state achieve a point in time when patients will note a sense of fullness or discomfort along with mucoid drainage that results in further irritation and soiling of underclothes.

Case Your patient is a 55-year-old man presenting with a strong, sudden need to urinate. He feels his bladder spasms and sometimes has involuntary loss of urine. The problem started several months ago and seems to be worsening. He is very upset because it sometimes happens during business meetings. His urinalyses were always normal. He takes no medications. Careful examination shows that he has urge incontinence. Bladder training and behavioral techniques were unsuccessful and you decided to introduce medication. Before suggesting a medication for this condition, you will tell him that this drug may have some side effects: dry mouth, difficulty in urination, constipation, blurred vision, tachycardia, drowsiness, and dizziness. Question This may happen because you will be prescribing what type of drug? Cholinergic Anticholinergic Epinephrine Antibiotic Botulinum toxin

Correct answer: Anticholinergic Explanation Urge incontinence is defined as involuntary loss of urine occurring for no apparent reason together with a feeling of urinary urgency (a sudden need or urge to urinate) that represents a hygienic or social problem to the individual. The most common cause of urge incontinence is involuntary and inappropriate detrusor muscle contractions. The drug you will suggest is anticholinergic (like Oxybutynin). It will relieve urinary and bladder difficulties, including frequent urination and urge incontinence by decreasing muscle spasms of the bladder, increasing the capacity of the bladder, and delaying the initial urge to void. It is a competitive antagonist of M1, M2, and M3 muscarinic acetylcholine receptors, and it can act as spasmolytic on bladder smooth muscle at higher doses. Anticholinergic side effects are dry mouth, difficulty in urination, constipation, blurred vision, tachycardia, drowsiness, and dizziness. Cholinergic drugs will cause slowing of the heartbeat and an increase in normal secretions. For this reason, patients who already have a problem with incontinence should not be advised to use these drugs. Epinephrine is not indicated in this patient. Epinephrine stimulates the ends of the sympathetic or inhibitory nerves of the bladder, with the effect of relaxation of the bladder muscles and the increase in tone and rate of contraction of the ureter. The secretion of urine is increased synchronously with the rise in arterial pressure. It will also cause overacting heart, palpitation, and vomiting. There is no need for antibiotics in a patient with normal urine analysis for a problem that lasts several months. Botulinum toxin is given as intradetrusor injection in patients who have failed pharmacological therapy. It has been shown to decrease episodes of urinary leakage by preventing the release of acetylcholine from presynaptic membrane. It is also indicated for urinary incontinence in patients with neurologic conditions (e.g., spinal cord injury, multiple sclerosis). It sometimes can cause urinary retention given as intradetrusor injections and occasionally headache, light-headedness, fever, abdominal pain, and diarrhea (not necessarily a direct result of Botox).

Case A 52-year-old Caucasian man presents with pain in his left lower extremity. The patient is well known to your practice. Upon further questioning, the patient describes doing construction work 3 months prior to presentation; he jumped onto his feet from an elevated height of 6 ft. Since this episode, he has noticed increasing left sided hip and knee pain. He describes the pain as radiating into the left groin and front middle thigh area. The pain is relieved with sitting and aggravated by walking and climbing up stairs. The patient denies any paresthesias, numbness, bowel/bladder dysfunction, fever, night sweats, or chills. Pertinent medical history includes alcoholism for the past 15 years; there have been 3 episodes of acute pancreatitis in the last 4 years. They required hospitalization. Physical examination reveals a patient with a new onset of a limp as well as restricted and painful active and passive joint movement of the left lower extremity. Question What is highest on your differential diagnosis at this time? Multiple myeloma Osteoporosis Avascular necrosis Osteoarthritis Rheumatoid arthritis

Correct answer: Avascular necrosis Explanation Avascular necrosis (AVN), which is sometimes also referred to as osteonecrosis, is osseous cell death resulting from vascular compromise. Common sites that are affected include the proximal or distal femoral head or even the ankle, shoulder, or elbow. Common causes include but are not limited to corticosteroid use, alcoholism, trauma, systemic lupus erythematosus, pancreatitis, gout, and sickle cell disease. The patient presented shows several risk factors for development of avascular necrosis, placing this very high on the most likely diagnosis. Occasionally, AVN will be asymptomatic and found incidentally on radiographic images. Symptoms will vary depending on the joint that is affected; pain in that joint is most common. Patients may experience groin pain that is worsened with weight bearing actions. This pain will usually progress to being present while at rest. On physical examination, patients who walk with a new limp are usually highly suspicious of having AVN progression and deterioration. Patients will have tenderness around that affected bone tissue; they will also have significant restriction as well as pain with active and passive joint movements. Neurologic deficits may also be present due to nerve compression from the necrosis. Severely advanced AVN can present with joint deformity or even muscle wasting. Multiple myeloma typically initially presents as spine, rib, or proximal long bone pain; it is more commonly in older adults, with the median age of presentation being 65 years. Many of these patients will present with signs and symptoms relating to pathology, including anemia, infections, kidney failure, spinal cord compression, or even hyper-viscosity syndrome. Osteoporosis is not common in this patient population group; therefore, it is much lower on the differential diagnosis. Its occurrence is higher in Caucasian women, with other risk factors such as tobacco or alcohol abuse, excess/deficiency in hormones, malignancy, and even some genetic disorders. Osteoporosis is very commonly asymptomatic until actual fractures occur from this pathology. Many times it will create back pain of varying degrees, depending if a fracture is present or not. Although the patient may have some signs of osteoarthritis on future imaging studies, it is most likely not the main cause of what has brought him in to see you now. Rheumatoid arthritis has an insidious onset, with characteristic morning stiffness, and it symmetrically affects small joints of the hands and feet.

Case A 77-year-old Caucasian woman is admitted to the hospital for episodes of dizziness when standing. Her symptoms have been present for approximately 4 months, and they have been progressively worsening. She has no dizziness when sitting or lying down. She does note some rapid heart rate associated with her dizzy spells. The dizziness becomes severe enough that she must sit down and is unable to proceed with walking, which is now significantly interfering with her functional activities. She denies associated chest pain, shortness of breath, or nausea. Past Medical History: Hypothyroidism, single episode of atrial fibrillation 2 years ago. Previous Surgeries: Hysterectomy, cholecystectomy, tonsillectomy. Allergies: No known drug allergies. Usual home medications: Aspirin 81 mg daily, Levothyroxine 88 mcg daily. Social History: Patient is married and has 2 adult children. She denies any history of smoking, alcohol use, or use of illegal drugs. Family History: Father had an acute MI at age 56, and died at age 84 of a ruptured aortic aneurysm. Mother died of CVA at age 85. She has 2 brothers. Oldest brother had lung cancer treated with thoracotomy, now alive at age 75. Younger brother is age 70 in good health. Two children are alive and well. Review of systems: Notable only for occasional cough, occasional headache, and mild anxiety. Vital signs: Temp 98.2°F, pulse 98 and regular, respirations 16, blood pressure 94/62 mm Hg (sitting, right arm). Question What is most likely to establish her diagnosis? Assessment for pallor of the skin and nails Assessment of skin turgor Blood pressures lying, sitting, and standing Cardiac auscultation Romberg test

Correct answer: Blood pressures lying, sitting, and standing Explanation Orthostatic blood pressure recordings (blood pressure when lying, sitting, and standing) can quickly identify the presence of orthostatic hypotension (low blood pressure upon standing). Orthostatic hypotension may be due to several causes: dehydration, blood loss, hypoglycemia, autonomic neuropathy, adrenal insufficiency, anemia, medications affecting blood pressure, and some cardiac conditions such as severe aortic stenosis or severe bradycardia. Orthostatic hypotension is defined as a drop in blood pressure—within 3 minutes of standing—of at least 20 mm Hg systolic and/or at least 10 mm Hg diastolic. This traditional definition was refined in 2011 by the American Academy of Neurology, which stated that for persons with supine hypertension, a systolic blood pressure drop of at least 30 mm Hg might be a more suitable measure of orthostatic hypotension. The AAN also defined "initial" orthostatic hypotension as a transient blood pressure decrease of at least 40 mm Hg systolic and/or 20 mm Hg diastolic within 15 seconds of standing, observed with continuous beat-to-beat blood pressure monitoring. Orthostatic hypotension should be distinguished from POTS (postural orthostatic tachycardia syndrome), which is manifested by elevated heart rate with standing or exertion without postural hypotension. Assessment for pallor of the skin and nails, assessment of skin turgor, and cardiac auscultation are all incorrect. While these may identify factors or conditions associated with orthostatic hypotension, they will not identify the positional drop in blood pressure that establishes the diagnosis of orthostatic hypotension. Pallor of the skin and nails may indicate anemia or acute blood loss, either of which can be associated with dizziness and orthostatic hypotension. Poor skin turgor can be a sign of dehydration, an underlying cause of orthostatic hypotension. Cardiac auscultation may identify aortic stenosis, bradycardia, or very rapid heart rhythm. Each of these may be an underlying cause of orthostatic hypotension. Dizziness or syncope with exertion such as walking is a classic symptom of severe aortic stenosis. The Romberg test is used to identify causes of ataxia (discoordination) related to position sense/proprioception, balance, or spatial coordination. Balance and spatial coordination are regulated by the vestibular function of the inner ear. Although ataxia such as staggering may be confused with dizziness, ataxia is unrelated to blood pressure abnormalities upon standing.

A 35-year-old man presents with right flank pain that has progressively worsened over the past 12 hours and is now radiating into his right groin and testicle. He currently rates the pain as an 8/10 and complains of some nausea but no vomiting. He denies ever having this type of pain previously. He states that he thinks he had some blood in his urine at his last void, but he did not notice any prior to that. The patient is unable to sit still during the interview and refuses the portion of the physical exam where CVA tenderness is assessed; he also refuses any palpation of the abdomen or testicle on the right side. Vital signs include a temperature of 99.2°F, BP is 156/84 mm Hg, RR is 12, and oxygen sat 99% on room air. Question What is the best study to assess this patient's condition? CT scan of abdomen and pelvis without contrast MRI of abdomen and pelvis Ultrasound of kidneys Kidney, ureters, bladder X-ray Intravenous urogram

Correct answer: CT scan of abdomen and pelvis without contrast Explanation This patient most likely suffers from urolithiasis. Such patients typically present with unilateral renal colic that often radiates to the ipsilateral groin or testis and hematuria. These patients generally cannot sit still secondary to the pain and discomfort. They can also exhibit guarding, nausea, and vomiting in some cases. A stone protocol (non-contrast) CT scan has become the standard initial workup of patients with suspected stone. This study is especially useful in the emergency department since it can quickly and effectively diagnose urolithiasis. A CT scan gives the most information (location and size of the stone, hydronephrosis, any anatomical variations, etc.) to the urologist who will likely see the patient. Stones are more common in men than women (3:1), and initial presentation is typically in the third or fourth decade of life. MRIs are used more often in assessing soft tissues; they are not typically useful in urolithiasis cases. Ultrasound of the kidneys can reveal hydronephrosis and cystic or solid lesions. Stones can sometimes be assessed as well, but small stones are occasionally not visible. A kidney, ureter, bladder X-ray will show approximately 90% of stones. Uric acid stones are not visible on plain film, and some stones are difficult to see because of size or location. A KUB is a viable option but not the best study to order. Intravenous urogram is most useful after a thorough bowel preparation, so a non-emergent study will typically yield more information than an emergent study. However, if a high-grade obstruction is suspected, emergent intervention with intravenous urogram may be warranted. These patients can have an elevated temperature and a CT scan can show high-grade obstruction, so that is still the best initial study.

Case A 23-year-old man presents with a 2-day history of burning urine. He also reports a slight purulent urethral discharge. He denies any fever, malaise, or chills. He smokes 1 pack of cigarettes daily and drinks socially; he has multiple sexual partners. On exam, his vitals are normal and lungs are clear; abdominal exam is unremarkable, without any renal angle or suprapubic tenderness, and external genitals reveal only slight urethral discharge. Labs show WBC of 6500/uL, and urinalysis has 5-10 WBC and 0 RBC. Gram stain of the urethral discharge shows neutrophils and intracellular gram-negative diplococci. Question Ico-delete Highlights What is the best treatment regimen for this patient? Ceftriaxone 250 mg IM plus azithromycin 1g PO each single dose Ofloxacin 400 mg BID for 7 days Trimethoprim-sulfamethoxazole for 3 days Doxycycline 100 mg daily for 7 days Ceftriaxone 125 mg IM single dose

Correct answer: Ceftriaxone 250 mg IM plus azithromycin 1g PO each single dose Explanation The correct response is ceftriaxone 250 mg IM plus azithromycin 1g PO each single dose. This patient has gonococcal urethritis, evidenced by the diplococci in the urethral discharge. The causative organism of gonorrhea is Neisseria gonorrhoeae, which is a gram-negative intracellular diplococcus. Gonorrhea treatment is complicated by the ability of N. gonorrhoeae to develop resistance to antimicrobials. A theoretical basis exists for combination therapy using two antimicrobials with different mechanisms of action (e.g., a cephalosporin plus azithromycin) to improve treatment efficacy and potentially slow the emergence and spread of resistance to cephalosporins. In addition, clinical trials have demonstrated the efficacy of azithromycin 1 g for the treatment of uncomplicated urogenital gonorrhea. Furthermore, those infected with N. gonorrhoeae frequently are coinfected with Chlamydia trachomatis; this finding has led to the long-standing recommendation that persons treated for gonococcal infection also be treated with a regimen that is effective against uncomplicated genital C. trachomatis infection, further supporting the use of dual therapy that includes azithromycin. Therefore, the use of ceftriaxone alone is not the best choice for treatment. It is most commonly seen in people ages 15-29. It is usually transmitted during sexual activity, and it has increasing incidence in homosexual men. Multiple sexual partners, unprotected intercourse, and anal sex are important risk factors. The incubation period of Neisseria gonorrhoeae is 2-8 days. Other than urethritis, it can also cause epididymitis, prostatitis, proctitis, cervicitis, vaginitis, and salpingitis. Pharyngeal infection is atypical, but it is not uncommon. Disseminated disease is associated with fever, rash, arthritis, and tenosynovitis. Chronic infection can cause urethral strictures, chronic salpingitis, infertility, and chronic prostatitis. Asymptomatic infection is quite common and can occur in men and women. Chronic cervicitis is an important reservoir of infection. Gram stain of the urethral discharge shows gram-negative diplococci inside neutrophils. The gold standard is a positive culture from any site, including the urethra, cervix, pharynx, and rectum. DNA probes can be used for urethral and endocervical specimens. Nucleic acid amplification tests are available for testing urine and urethral specimens. Erythromycin base, erythromycin ethylsuccinate, levofloxacin, ofloxacin, or doxycycline are acceptable alternatives to azithromycin for nongonococcal urethritis (NGU) only, but there is a distinct possibility of noncompliance with a 7-day course. A 3-day course of trimethoprim-sulfamethoxazole is suitable only for uncomplicated cystitis in women, and it would be useless in urethritis in men.

A 45-year-old man presents with left-sided headaches. He describes the pain as sharp, almost stabbing in nature. Headaches last 30-60 minutes, he rates the pain at 9/10, and he finds it difficult to sit still during the episodes. He notes that, during this time, he also has tearing of his left eye. The headache has been occurring daily for the past 15 days. He had a similar episode occur about 2 months ago, and the headaches resolved spontaneously. Question What is the most likely diagnosis? Brain tumor Migraine headache Tension headache Cluster headache Transient ischemic attack

Correct answer: Cluster headache Explanation Cluster headaches have a classic presentation of severe unilateral pain that lasts anywhere from 15 minutes to 3 hours. These "clusters" may occur once or multiple times daily, and they tend to occur daily for weeks to months. Attacks may be accompanied by nasal congestion, lacrimation, rhinorrhea, conjunctival erythema, miosis, ptosis, or anhidrosis on the affected side. Following a series of attacks, patients typically have remission of several weeks or even months. Brain tumors are associated with headaches, but they typically do not follow the above pattern. Headaches occurring with brain tumors may become more severe over time, and they usually do not resolve spontaneously. Migraine headaches tend to be associated with unilateral pain, but the attack typically lasts 4-72 hours. Tension headaches are less severe than cluster headaches. Patients usually describe the pain as a bilateral tightness sensation. The headaches are not usually associated with other symptoms. Transient ischemic attack mimics the symptoms of stroke. Symptoms usually last 1-2 hours, but most last less than 24 hours.

A 62-year-old man presents with a 2-month history of worsening fatigue and shortness of breath. He has a past medical history of emphysema attributable to his 85 pack-year smoking history. The patient complains of nearly passing out while climbing the stairs in his house. He tells you that he feels like his heart races. He reports chronic shortness of breath and cough, but he now he feels like his dyspnea is dramatically worse; he can no longer sleep in his bed. He has been trying to sleep propped up in a chair at night. He is also experiencing fatigue. He has gained about 15 pounds, and he notes that he can no longer lace up his shoes. He denies fever, chills, and chest pain. His cough produces some mucus, but no hemoptysis. His vitals are shown in the table. Weight 212 lb Height 69" Body mass index 31.3 Pulse 108 Blood pressure 140/88 Temperature 98.2°F Pulse oximetry 88% On physical exam, you see a man in mild respiratory distress; he is sitting upright and leaning forward, and he uses accessory respiratory muscles for breathing. The exam is significant for reduced air movement and mild rales bilaterally in the lungs; distended neck veins; mild tachycardia with prominent P2; lower extremity edema; and right upper quadrant abdominal tenderness with hepatomegaly. Question Based upon this patient's history and physical, what is the most likely diagnosis? Cor pulmonale Hepatitis Lung abscess Pneumonia Pulmonary embolism

Correct answer: Cor pulmonale Explanation With a history of worsening primary pulmonary disease (chronic obstructive pulmonary disorder or COPD) as well as the development of right-sided heart failure symptoms, this patient likely has a diagnosis of cor pulmonale, which is also known as pulmonary heart disease. In mild cases, cor pulmonale typically presents with just the symptoms of the lung disease; however, in moderate-to-severe presentations, heart failure symptoms are prominent. Due to increased vascular resistance in the pulmonary vasculature, the right ventricle hypertrophies and eventually becomes unable to push enough cardiac output through the pulmonary circulation and fluid "backs up." This explains this patient's distended neck veins (jugular venous distention), ankle edema, and weight gain. The pulmonic component of S2 heart sounds (P2) also becomes more prominent. Hepatitis could present with right upper quadrant tenderness and hepatomegaly in acute cases, but it would not explain all of this patient's cardiovascular and pulmonary signs and symptoms. Typically, chronic hepatitis does not present with tenderness or enlargement of the liver. Lung abscess is most often a complication of a pulmonary infection, such as pneumonia or tuberculosis. An abscess is difficult to distinguish from infection, but neither would produce the cardiovascular symptoms that the patient has. Pneumonia could certainly present with dyspnea, fatigue, and respiratory distress; however, this patient's lack of fever, report of weight gain, and cardiovascular findings all go against a diagnosis of pneumonia. A pulmonary embolism (PE) could produce dyspnea and tachycardia. Most often, the symptoms are quite acute; they include chest pain without the right-sided heart failure symptoms (peripheral edema, jugular venous distention, and hepatomegaly). Because this patient is a smoker, he could also be at risk for a PE, but a current diagnosis of PE does not adequately explain all of his current symptoms and exam findings.

A 32-year-old woman presents with a 3-day history of irritation, burning, itching, and redness of both eyelids. She denies fever, visual changes, and photophobia. On physical examination, you note the presence of scales clinging to the eyelids bilaterally. Question Ico-delete Highlights What is the proper management in this case? Daily cleaning with a damp cotton applicator and baby shampoo Short-term oral antibiotic therapy for 7 days Short-term oral corticosteroid therapy for 14 days Topical corticosteroid eye drops for 10 days Prompt ophthalmologist referral

Correct answer: Daily cleaning with a damp cotton applicator and baby shampoo Explanation The scenario presented above depicts a patient with anterior blepharitis, which is a common disorder seen in primary care; it typically consists of a recurrent bilateral inflammation of the lid margins that involves the eyelid skin, eyelashes, and associated glands. Commonly, the underlying cause is seborrhea, which usually originates in the scalp, eyebrows, or ears. Sometimes, anterior blepharitis can be ulcerative, and the origin in the presented case is staphylococci. Anterior blepharitis can typically be resolved and controlled by cleaning the affected areas daily using a damp cotton applicator, warm water, and a baby shampoo mixture. The object of the daily cleaning is to remove the visible scales as efficiently as possible. None of the other listed options are an appropriate treatment plan for anterior blepharitis. Patients can also be diagnosed with what is known as posterior blepharitis, which is an inflammation of the meibomian glands of the eyes. It is usually staphylococcal in origin, and it typically presents with significantly worse signs and symptoms, such as hyperemic lids, the presence of telangiectasias, inflammation of the gland or their orifices, or even abnormal secretions; tears may be described as being frothy or greasy. More significant cases of posterior blepharitis can lead to conjunctivitis, hordeola, chalazions, eyelash trichiasis, or even corneal vascularization and thinning. Treatments for posterior blepharitis may consist of long-term oral antibiotic therapy, short-term topical steroids, or short-term topical antibiotics eye drops; if significant complications are evident, an ophthalmologist referral is indicated.

A 14-year-old boy presents with worsening shortness of breath; it most often occurs when he plays soccer. He often awakens in the middle of the night due to "attacks": he starts to feel anxious because he feels like he cannot breathe, and he experiences chest tightness. He suffers from a dry cough, especially after playing sports. His mother has put a humidifier in his room and has him use his sister's inhaler, which seems to help temporarily; the boy has been using it 5-6 times daily. The family history is significant for asthma in his sister, father, and three other paternal relatives. The patient and his mother are not aware of any allergies. He denies fever, chills, and chest pain. In between "attacks," he feels well and normal. The patient's past medical history is noncontributory. There are no known medical conditions; he has no drug allergies, and he has not had any surgeries. Other than the aforementioned inhaler, he does not take any medications. Question In addition to his own albuterol inhaler, what medication should be prescribed for this patient? Burst of oral prednisone Daily inhaled salmeterol Daily low-dose inhaled budesonide Daily oral zileuton Subcutaneous injections of omalizumab

Correct answer: Daily low-dose inhaled budesonide Explanation This patient is exhibiting persistent asthma symptoms. Using the stepwise approach to treatment, the second step (after using a short-acting beta-agonist, such as albuterol) is to add a low-dose inhaled corticosteroid. Alternatives include cromolyn, a leukotriene receptor antagonist, nedocromil, and theophylline. For this patient, daily low-dose inhaled budesonide would be most appropriate. He should then be monitored for response; if necessary, the medication should be adjusted. The patient and his mother should receive patient education on asthma, the use of peak flow meters, and the proper use of medications. A burst of oral prednisone can be useful in the management of an acute exacerbation of asthma, but its use is discouraged due to the side effects of systemic steroids. This patient is not in acute distress and should start with inhaled steroids, which pose fewer side effects than oral/systemic steroids. Daily-inhaled salmeterol, a long-acting beta-agonist (or LABA), should be added to an inhaled corticosteroid if low-medium doses of the inhaled steroid alone are unable to control symptoms. The salmeterol is not recommended as a stand-alone therapy, and it should only be used with other asthma control medications. Daily oral zileuton, a 5-lipoxygenase inhibitor, is extremely expensive and is dosed 4 times daily. It should not be recommended initially when the preferred agent (low-dose inhaled corticosteroid) has not yet been tried. Additionally, compliance in teens can be difficult, so a 4-times-daily medication is not ideal. Subcutaneous injections of omalizumab, an immunomodulator, are an option in step 5 and 6 patients who have failed to achieve symptom control with multiple and high-dose medications. Omalizumab is not appropriate in this patient's case.

A 35-year-old woman notices a change in the appearance of a mole on her neck. Physical examination reveals that the lesion is an irregular nodular superficial mass with a variegated appearance. Biopsy demonstrates a primary malignant tumor. Question What characteristic of the malignant skin lesion is most predictive of the patient's long-term prognosis? Sharpness of border Diameter Color Variation Depth Asymmetry

Correct answer: Depth Explanation The lesion is a malignant melanoma. Melanomas can develop either de novo or in an existing mole. Sunlight exposure is a significant risk factor, and fair-skinned individuals are at increased risk of developing melanoma. The most significant factor for long-term prognosis is the depth of the lesion since the superficial dermis lies about 1 mm under the skin surface; penetration to this depth is associated with a much higher incidence of metastasis than is seen with a more superficial location. The diameter of the lesion is much less important than the depth since one form of melanoma (superficial spreading) can still have a good prognosis—despite large size—if it has not extended to the depth of the superficial dermal lymphatic bed. The degree of color variation does not have prognostic significance once melanoma is diagnosed. The asymmetry of a lesion and irregularity of the border of a mole-like lesion are good clues to potential malignancy, but they do not affect prognosis once a melanoma is diagnosed.

You are evaluating a 24-year-old woman for bilateral eye pain. She describes red itching irritated eyelids for several weeks. She states she has had "several bouts" of similar symptoms over the last few years. Exam is consistent with blepharitis. She does not wear contacts, and she occasionally wears eye makeup. She denies any other infectious complaints. Question What is the most appropriate treatment? Start the patient on ophthalmic antibiotics. Start the patient on ophthalmic steroids. Discuss good eye hygiene and eyelid scrubbing. Refer to an ophthalmologist for surgical correction. Apply beta blocker to decrease intraocular pressure.

Correct answer: Discuss good eye hygiene and eyelid scrubbing. Explanation The correct answer is that you should discuss good eye hygiene and eyelid scrubbing with the patient. Blepharitis is chronic inflammation of the eyelids. Patients often present with irritation of the lid margins that may produce minor crusting, a gritty sensation when blinking, eye itching and redness but a full preservation of vision. The majority of patients have recurrent bouts of inflammation, which can be resolved by eyelid washing and avoidance of eye makeup. The treatment of blepharitis rarely requires antibiotics. While it is possible to have a secondary infection—typically staph aureus—there is no evidence of a secondary infection in our patient. Initiation of ophthalmic steroids should be done under the guidance of an ophthalmologist. Additionally, there is no benefit of an ophthalmic steroid in the treatment of blepharitis. The majority of cases of uncomplicated blepharitis do not need to be referred to an ophthalmologist unless the diagnosis is uncertain or there are other complicating factors. Additionally, there is no surgical correction for uncomplicated blepharitis. Our patient does not have signs and symptoms of increased intraocular pressure.

A 7-year-old boy presents for evaluation (recommended by his teacher) with a 6-month history of behavioral problems in school. Although the boy tests at grade level, he seems to make careless mistakes on schoolwork, has trouble paying attention to instructions, does not finish homework, and often loses his homework, pencils, and books. The boy is often seen fidgeting at his desk. He blurts out answers and has difficulty waiting his turn. The parents agree that they have seen similar traits at home (e.g., forgetting to do daily activities and easy distractibility) for several years. They view him as a happy, bright boy who is very active. He has had normal vision and hearing screenings. The father reports that he was very similar to his son when he was a child and still struggles with focus and concentration as an adult. The parents deny any major changes in the family situation. They would like medical help to improve their son's performance in school. The boy has been seen regularly for his well-child exams and has always met milestones. Today, he quickly moves about the exam room looking at a book for a few moments, then to the window, then interrupting his parents. Question What has been identified as the primary neurotransmitter responsible for this child's likely disorder? Acetylcholine Dopamine Gamma-aminobutyric acid (GABA) Glutamate Histamine

Correct answer: Dopamine Explanation This child meets the criteria for attention deficit hyperactivity disorder (ADHD), a disorder most associated with dopamine abnormality. Medications such as methylphenidate, a central nervous system stimulant, are considered a first-line therapy for ADHD; the drugs work by regulating dopamine and norepinephrine levels. Acetylcholine was the first neurotransmitter to be discovered. It has several roles in central and peripheral neurologic actions and within the parasympathetic system. It is not directly linked to ADHD. Gamma-aminobutyric acid (GABA), an amino acid, plays a major inhibitory role as a neurotransmitter. Some anticonvulsants are thought to work through the GABA receptors. GABA has some links to schizophrenia and panic disorder but is not currently thought to have a major role in ADHD. Glutamate, derived from glutamine or aspartate, is the major excitatory neurotransmitter in the central nervous system. Much attention has been focused on N-methyl-d-aspartate (NMDA), a glutamate receptor. Overstimulation of NMDA receptors is thought to lead to excitotoxicity of the cells and play a role in neurodegenerative disorders (e.g., Alzheimer's and Huntington's diseases). Glutamate is not directly linked to ADHD. Histamine is commonly associated with its role in immune/inflammatory responses and gastric acid secretion, but it is also a neurotransmitter associated with stimulated wakefulness, suppressed appetite, and enhanced cognition. It is not associated with ADHD.

Case A 40-year-old man with no significant past medical history presents with a 2-day history of alternating fever and rigors, diaphoresis, fatigue, and a productive cough. He admits to mucoid sputum of moderate quantities. He denies a history of smoking, alcohol use, recent travel, or sick contacts. He further denies chest pain, palpitations, hemoptysis, rashes, abdominal pain, nausea, vomiting, or diarrhea. On physical exam, he is found to be tachypneic and was observed to be intermittently coughing. The pulmonary exam was notable for bronchial breath sounds over the right anterior fourth, fifth, and sixth intercostal spaces. A chest radiograph revealed a right middle lobe consolidation. Question What additional physical exam finding would be consistent with this patient's most likely diagnosis? Cheyne-Stokes respiration Bradycardia Decreased tactile fremitus Decreased whispered pectoriloquy Dullness to percussion

Correct answer: Dullness to percussion Explanation This patient is most likely presenting with community-acquired pneumonia (CAP). Most patients with CAP experience an acute or subacute onset of fever, cough with or without sputum production, and dyspnea. Other common symptoms include sweats, chills, rigors, chest discomfort, pleurisy, hemoptysis, fatigue, myalgias, anorexia, headache, and abdominal pain. Common physical findings include fever or hypothermia, tachypnea, tachycardia, and arterial oxygen desaturation. Patients may appear acutely ill. Chest examination often reveals inspiratory crackles and bronchial breath sounds. Dullness to percussion may be observed if lobar consolidation or a parapneumonic pleural effusion is present. Increased tactile fremitus, bronchophony, egophony, and whispered pectoriloquy occur over consolidated lung tissue. Radiographic findings range from patchy airspace opacities to lobar consolidation with air bronchograms to diffuse alveolar or interstitial opacities. Cheyne-Stokes respirations are periods of deep breathing alternate with periods of apnea. It occurs normally in children and aging people, and pathologically in heart failure, uremia, drug-induced respiratory depression, and brain damage.

Case A 22-year-old woman presents for evaluation of knee pain. The patient appears in moderate distress and is unable to fully bear weight as she moves to the exam table. Her physical exam reveals a swollen, tender left knee. She is tender at the lateral femoral condyle, lateral tibial plateau, and tibiofemoral joint line. Lachman and anterior drawer tests are positive. Posterior drawer, Apley grind, patellar grind, and McMurray tests are negative. There are no sag or apprehension signs. Valgus and varus stress tests are normal. An X-ray is obtained and shows no fractures. Question What inciting event is most likely in this patient's history? Dashboard injury to knee during a motor vehicle accident Fever and prior Neisseria gonorrhoeae infection Frequent squatting and climbing stairs Sudden landing after jumping during basketball Lateral blow to knee

Correct answer: Sudden landing after jumping during basketball Explanation This patient's physical exam findings demonstrate a deficiency in the anterior knee, specifically the anterior cruciate ligament (ACL). ACL injuries most often occur acutely while playing sports, such as basketball, skiing, and tennis; however, they can occur with other mechanisms as well. Female gender is a risk factor. The patient may give a history of hearing a "pop", along with acute pain as well as buckling or instability of the knee. Of the choices listed, sudden landing after jumping during basketball is the most likely to produce an ACL injury. A dashboard injury to the knee during a motor vehicle accident could produce several injuries, but it would be most likely to produce a posterior dislocation and disrupt the posterior cruciate ligament (PCL). A PCL injury would likely yield physical exam findings of a positive posterior drawer and sag sign (in which the anterior tibia sags posteriorly). Fever and prior Neisseria gonorrhoeae infection with knee pain suggests septic arthritis as the cause. Septic arthritis would produce swelling and tenderness, but it would not be expected to demonstrate instability, such as this patient's positive anterior drawer and Lachman tests. Frequent squatting and climbing stairs can contribute to knee pain, and would be suggestive of a patellofemoral syndrome or possibly osteoarthritis (although the latter is unlikely in a 22-year-old patient). If this patient's knee pain had been caused by squatting and stair-climbing, her physical exam might have demonstrated a positive apprehension test and patellar grind test. A lateral blow to the knee is associated with disruption of the medial and/or lateral collateral ligaments. Here, we would expect the physical exam to have a positive valgus and/or varus stress test. A lateral knee injury would not typically cause ACL injury.

An 82-year-old man with a past medical history of hypertension, dyslipidemia, type 2 diabetes, and chronic kidney disease is being evaluated for progressive exercise-induced fatigue and shortness of breath over the last year. He also admits to more recent chest pain and lightheadedness, both of which occur with ambulation. He denies cough, fever, chills, lower extremity edema, or abdominal complaints. The physical exam revealed a narrow pulse pressure following blood pressure assessment. His cardiac exam noted a laterally displaced point of maximal impulse, as well as a mid-systolic ejection murmur that is low-pitched, rough, rasping in character, and loudest in the second right intercostal space. This murmur radiates to the bilateral carotid arteries. His peripheral vascular exam demonstrated a delayed peak of his radial pulsations. Question What diagnostic procedure would be best for this patient? EKG Echocardiography Lateral chest X-ray Cardiac catheterization Immediate coronary angiography

Correct answer: Echocardiography Explanation This patient's presentation is consistent with aortic stenosis (AS). In most patients with severe AS, there is left ventricular hypertrophy. The key findings on transthoracic echocardiography are thickening, calcification, reduced systolic opening of the valve leaflets, and left ventricular hypertrophy. Echocardiograms are also useful in identifying coexisting valvular abnormalities, differentiating valvular aortic stenosis from other forms of left ventricular outflow obstruction, and for measurement of the aortic root and proximal ascending aortic dimension. EKG can show the hypertrophy of the heart caused by the aortic stenosis and any other conduction abnormalities, but it will not make the diagnosis of aortic stenosis. Aortic valve calcification may be discernible in the lateral view of a chest X-ray; the absence of valvular calcification in an adult suggests that severe valvular AS is not present. Cardiac catheterization for invasive assessment of aortic stenosis is performed infrequently but can be useful when there is a discrepancy between the clinical and Doppler echocardiographic findings. It is useful for specific types of patients: patients with multivalvular disease, especially regarding operative treatment young, asymptomatic patients with non-calcific congenital aortic stenosis patients in whom it is suspected that the obstruction to LV outflow may not be at the level of the aortic valve Coronary angiography is indicated to detect or exclude CAD in appropriate patients with severe stenosis who are being considered for surgery.

Case A 47-year-old Caucasian woman, previously healthy, presents for evaluation of a 4-week history of dyspnea. Her symptoms started approximately 1 month ago when on vacation in Colorado. She initially attributed these symptoms to the altitude; however, upon return to her hometown, she continued to have shortness of breath with mild activity, such as walking more than 100 feet, walking upstairs 1 flight, vacuuming, or sweeping. Her symptoms resolve with rest. She also reports mild exertional chest tightness and easy fatigability. She denies paroxysmal nocturnal dyspnea, orthopnea, edema, palpitations, and syncope. Her past medical history includes the usual childhood illnesses, no previous surgeries, and no known allergies. She takes a daily multivitamin and occasional Tylenol for headache. She is a non-smoker, rarely uses alcoholic beverages, and denies use of illegal or illicit drugs. Physical exam shows temp 96.9°F, pulse 80 and regular, respirations 16, and BP 136/82. O2 sat is 96% on room air. The patient is an alert Caucasian woman in no acute distress, with no obvious jugular venous distention; non-labored respirations; lung fields clear to auscultation and percussion; and no rhonchi, rales, or wheezes. Heart shows RV heave present; normal S1 with fixed, split S2 with prominent P2 component; and grade II/VI systolic murmur at the left upper sternal border at the second intercostal space. The remainder of a complete physical examination is within normal limits. CBC and BMP are unremarkable. Free T4 and TSH are within normal limits. EKG shows normal sinus rhythm with right ventricular hypertrophy, right atrial enlargement, and right axis deviation. There is an RSR in leads v1 and v2. Question What severe complication sometimes develops in patients with this clinical presentation and may require treatment with combined heart-lung transplantation? Cardiac rupture Cor pulmonale Eisenmenger syndrome Mitral valve rupture Saddle embolus

Correct answer: Eisenmenger syndrome Explanation The correct answer is Eisenmenger syndrome, a condition in which a long-standing left-to-right cardiac shunt caused by congenital heart disease (typically an atrial septal defect or ventricular septal defect) converts (often in association with pulmonary hypertension) into a cyanotic right-to-left shunt. ASD or VSD is most often associated with a left-to-right cardiac shunt, but in the presence of extremely elevated right heart pressures (which can occur with severe pulmonary hypertension) that exceed the left heart pressures, that shunt will convert to a right-to-left shunt. A right-to-left shunt is a "cyanotic" shunt because it results in venous (deoxygenated) blood from the right side of the heart being diverted back to the systemic circulation. The only truly definitive treatment of Eisenmenger syndrome is lung transplantation with shunt closure or combined heart and lung transplantation. (Atrial septal defect alone does not require treatment with a combined heart-lung transplant. Atrial septal defect might be treated conservatively with observation or with a surgical closure of the defect. Only once the patient has developed the complication known as Eisenmenger syndrome would a combined heart-lung transplant be considered.) Cardiac rupture is incorrect. Cardiac rupture is usually a consequence of myocardial infarction or thoracic trauma. Cor pulmonale is incorrect. Cor pulmonale is usually described as the failure of the structure and function of the right ventricle caused by an underlying primary disorder of the respiratory system. Pulmonary hypertension is a frequent cause of cor pulmonale. COPD and pulmonary emboli are also common causes of cor pulmonale. While the patient in the case may have or may develop cor pulmonale, that condition would not be treated with a heart-lung transplant. Mitral valve rupture is incorrect. Rupture of the papillary muscle of the mitral valve is most commonly due to myocardial infarction or chest injury. Rupture of the chordae tendineae of the mitral valve may be spontaneous or may be due to trauma, infective endocarditis, myxomatous disease, rheumatic heart disease, or may be secondary to displacement of a mechanical circulatory device. Saddle embolus is incorrect. Saddle embolus refers to a large pulmonary embolus that straddles the bifurcation of the main pulmonary artery. Pulmonary emboli, including saddle emboli, most often occur as a complication of deep venous thrombosis. Risk factors for deep venous thrombosis, and hence pulmonary emboli, include heart failure, surgery, prolonged immobility, clotting disorders, and certain cancers.

A 70-year-old woman with a history of hypertension, hyperlipidemia, and myocardial infarction presents with a 3-day history of shortness of breath at rest. She has shortness of breath with minimal activity such as walking short distances. Additionally, she complains of orthopnea and paroxysmal nocturnal dyspnea. She denies cough, fever, chills, nausea, abdominal pain, vomiting, diarrhea, or rashes. Upon physical examination, the patient appears short of breath and has conversational dyspnea. She is tachycardic and diaphoretic, and extremities are cool. Cardiovascular exam shows a decreased S1, S3 gallop, and laterally displaced PMI. There is no jugular vein distention (JVD) noted; however, 2+ pitting edema of the lower extremities to mid-calf level is evident. Pulmonary exam shows bibasilar rales, dullness to percussion, and expiratory wheezing noted. Question What diagnostic test result would be most useful in differentiating a cardiac cause from a non-cardiac cause of this patient's presentation? Hyponatremia on a basic metabolic profile Elevations of T3 and T4 on thyroid assay Sinus arrhythmia and low voltage on EKG Pulmonary congestion pattern on the chest X-ray Elevations of B-type natriuretic peptide

Correct answer: Elevations of B-type natriuretic peptide Explanation This patient's presentation is consistent with congestive heart failure. Rapid measurement of B-type natriuretic peptide (BNP) or its precursor, N-terminal proBNP (NT-proBNP), can aid clinicians in differentiating between cardiac and non-cardiac causes of dyspnea. The major source of plasma BNP is the cardiac ventricles, and the release of BNP appears to be in direct proportion to ventricular volume and pressure overload. BNP levels greater than 80 pg/mL have a specificity greater than 95% and a sensitivity greater than 98% in the diagnosis of heart failure; higher BNP levels correlate with higher sensitivity and specificity of the presence of heart failure. Electrolyte levels, thyroid assessments, EKG, and chest X-ray findings are important diagnostic modalities useful in the evaluation of CHF, but they cannot readily differentiate cardiac versus non-cardiac causes of this patient's presentation to the extent that B-type natriuretic peptide can.

A 53-year-old man presents with increased difficulty swallowing and occasional regurgitation of his meals. His symptoms have been occurring with greater frequency and severity over the last 4 months. He also gets some shortness of breath but attributes that to his weight and lack of physical activity. His past medical history is remarkable for chronic heartburn, which he treats intermittently with over-the-counter antacids. He takes no regular medications and he has no allergies. He has not had any surgeries. He is a smoker, but he denies use of alcohol and drugs. He works as a building inspector and he lives with his wife and children. The patient is obese, but the rest of his physical exam is normal. Blood tests, electrocardiogram, and chest X-ray are done in the clinic; they are normal. He is referred for endoscopy, and esophageal biopsy shows specialized intestinal metaplastic cells (of columnar epithelium). Question What is the medication of choice for this patient's condition? Alendronate Dicyclomine Esomeprazole Famotidine Simethicone

Correct answer: Esomeprazole Explanation This patient is presenting with Barrett's esophagus, which is a type of chronic esophagitis in which the normal squamous epithelium is replaced by columnar epithelium. Barrett's esophagus is a complication of chronic gastroesophageal reflux disease (GERD) and can develop into esophageal adenocarcinoma. Use of proton pump inhibitors (PPIs), such as esomeprazole, reduces the risk of cancer. In order to monitor for the development of cancer, a routine endoscopy should be periodically performed in patients with known Barrett's esophagus. Alendronate is a bisphosphonate; it is used for both treatment and prevention of osteoporosis. Alendronate can cause multiple gastrointestinal adverse effects, including esophagitis and esophageal ulcerations and strictures. This medication could dramatically worsen this patient's current condition. Dicyclomine is an anticholinergic medication that relaxes smooth muscle. It is used for spasms associated with irritable bowel syndrome. It does not have a role in treating GERD or Barrett's esophagus. Famotidine antagonizes H2 histamine receptors (H2 blocker). It is used in treating duodenal and gastric ulcers as well as GERD. H2 blockers are usually considered less potent than PPIs. PPIs are the treatment of choice for Barrett's esophagus. Simethicone is sold in various over-the-counter products for flatulence. It has no role in treating GERD or Barrett's esophagus.

Case A 32-year-old woman presents for follow-up after being seen in the emergency department 2 days prior. Her mother brought her in after witnessing the patient having a seizure. While in the ED, the patient was observed having another seizure. She appeared to be shaking with asynchronous movement of her right and left arms. She remained conscious throughout the episode. EEG monitoring was negative for any seizure-like activity. The patient was also seen in the ED 3 months ago following a sexual assault. The patient notes poor sleep and difficulty motivating herself to complete her daily activities. On examination, the patient appears tearful. Neurological examination is normal. Question What is the most appropriate medication for this patient? Levetiracetam Lamotrigine Lorazepam Fluoxetine Oxycodone

Correct answer: Fluoxetine Explanation The correct response is fluoxetine. The patient's history describes seizures, but the episode observed in the ER is not consistent with a seizure; the lack of synchronous movement and absence of EEG findings shows that this patient is not having seizure-like activity. This, paired with her recent history of sexual assault, is suggestive of conversion disorder. Conversion disorder is a somatoform disorder in which psychological stress is converted into physical symptoms. The recommended treatment is therapy; if medications are used, antidepressants or antianxiety medications are recommended. The patient appears to be exhibiting signs of depression; fluoxetine would be an appropriate choice. The patient should undergo therapy with close follow-up, in addition to taking fluoxetine. Levetiracetam and lamotrigine are anticonvulsant medications. The patient does not exhibit any clinical signs of actual seizure activity. Lorazepam has many uses, including antiepileptic and sedative effects. Sedative medications are often overused in patients with somatoform disorders. Its use should be avoided. The patient is not currently complaining of pain, so oxycodone should not be prescribed. Patients with somatoform disorders are at higher risk of drug dependence and abuse due to the potentially long duration of their condition. Addictive medications should be avoided in this population.

A 56-year-old man presents for a routine follow-up regarding his positive HIV status. He is compliant with his medications and has been feeling well. In addition to his antivirals, he takes daily trimethoprim/sulfamethoxazole for Pneumocystis jirovecii pneumonia (PCP) prophylaxis. Question What detail of his history would warrant the prophylaxis? CD4 cell count <400 History of previous PCP infection CD4 cell count >200 History of any previous pneumonia infection HIV viral load >100,000 copies/mL

Correct answer: History of previous PCP infection Explanation History of previous Pneumocystis jirovecii pneumonia (PCP) infection is the correct answer. Patients who are HIV positive should take antibiotic prophylaxis for PCP if they have a history of PCP infection, if their CD4 cell count is below 200, or if they have evidence of immunodeficiency, such as oral candidiasis. Prophylaxis for PCP is typically daily trimethoprim/sulfamethoxazole double strength, but it can also be done with daily dapsone or monthly aerosolized pentamidine. If patients are on antiretroviral therapy and their CD4 cell count remains above 200 for at least 3 months, then they can stop prophylaxis. If their CD4 cell count drops below 200 again, they need to restart prophylaxis. CD4 cell count <400 is not the correct answer. Patients who are HIV positive need to be on PCP prophylaxis for various reasons, one of which is a CD4 count <200. A CD4 cell count between 200 and 400 would not warrant prophylaxis until the point that the count drops below 200. CD4 cell count >200 would actually be a reason NOT to have the patient on PCP prophylaxis, as one of the criteria to start prophylaxis is a CD4 cell count below 200. In addition, when a patient has been on prophylaxis and antiretrovirals and their CD4 cell count is over 200 for 3 months, they can stop the prophylaxis. History of any pneumonia infection is not the correct answer. Only patients who have had previous PCP infections need to be treated with prophylaxis for PCP. Having had other types of pneumonia does not warrant PCP prophylaxis. HIV viral load >100,000 copies/mL warrants antiretroviral treatment of HIV, but PCP prophylaxis is not given based on viral load. Rather, it is given based on CD4 cell count, previous PCP infection, or evidence of immunodeficiency.

A 48-year-old Caucasian woman presents due to feeling like she is losing her mind. She wants some tests done. Upon further questioning, she reports she is having multiple episodes daily in which she suddenly becomes very hot, flushed, and diaphoretic. These episodes last about 1 minute, then resolve. She has not measured a fever. The patient reports that the episodes occur during the day and at night, causing her to wake up drenched in sweat. As a result, her sleep has been poor, and she feels fatigued and irritable at both work and home. She has noticed these symptoms for about 2 months, and they seem to be increasing in severity. The patient has not had a period for 3 months; she recently did a home pregnancy test, which was negative. Prior to that, she had regular menses. This patient denies weight changes, palpitations, cold intolerance, bowel changes, as well as changes in her nails, skin, and hair. Although she admits irritability, she denies anxiety, depressed mood, and suicidal ideation. Her family history is remarkable for diabetes in her maternal grandfather and hypertension in her father. She is a G4P3Ab1. She denies any major psychosocial stressors recently. She drinks alcohol rarely, and she denies use of other drugs. Vitals and a urine specimen for hCG were obtained prior to the physical exam. Weight 168 lb Height 65" Pulse 72 Blood pressure 120/82 Temperature 98.2°F Urine hCG Negative Complete screening physical exam is normal, with normal sexual development and absence of hirsutism and acne. Question Ico-delete Highlights What diagnostic study result is most consistent with the suspected diagnosis? Decreased free thyroxine (free T4) Decreased luteinizing hormone (LH) Increased estradiol Increased follicle stimulating hormone (FSH) Increased testosterone

Correct answer: Increased follicle stimulating hormone (FSH) Explanation This patient would be expected to have increased follicle stimulating hormone (FSH) levels. Based on the history and physical, her diagnosis is perimenopause, with a classic history of vasomotor symptoms (hot flashes/flushes and night sweats). Perimenopause is defined as the time prior to menopause in which the woman's ovarian hormone output begins to decline, often beginning in the mid-40s. Menopause is defined at the point in time in which the patient has been amenorrheic for 12 months. Postmenopause occurs after that. The average age for menopause is just over 51 years of age. With her history, a negative pregnancy test, and normal physical exam at age 48, the diagnosis of perimenopause can actually be made without diagnostic studies. If tests are done, however, it is recommended to check the "upstream" hormones from the pituitary (follicle-stimulating hormone and/or luteinizing hormone) rather than the ovarian hormones themselves (estradiol and progesterone). FSH elevation is the first measurable hormone evidence of menopausal ovarian failure. With the physiologic feedback mechanism, both FSH and LH rise in response to a low ovarian output of inhibin, which would normally suppress FSH in a younger woman. A decreased free thyroxine (free T4) level would indicate hypothyroidism. Although thyroid function should be considered in women having menstrual changes and fatigue, this patient denies other symptoms of hypothyroidism (weight gain, dermatologic changes, cold intolerance, constipation, and depressed mood). Hypothyroidism is not associated with hot flashes and night sweats. Decreased luteinizing hormone (LH) would be expected in situations in which there is primary pituitary or hypothalamic dysfunction or suppression, such as with the administration of hormonal contraceptives. Eating disorders and malnutrition may also cause low LH levels. This patient presentation does not indicate any of these disorders. In perimenopause, and more dramatically in menopause, estradiol levels decrease. An increased estradiol level would not be expected in this patient. Increased testosterone levels are most often seen in women due to polycystic ovarian syndrome (PCOS); they can also be seen in women with primary tumors of the ovaries, in which androgen production is increased. PCOS is characterized by menstrual changes consistent with anovulation (amenorrhea, oligomenorrhea, and history of infertility), hyperandrogenism (e.g., hirsutism and acne), and characteristic polycystic ovaries are seen on ultrasound. PCOS may be associated with obesity and insulin resistance or diabetes. The ovarian tumors are rare. This patient does not have a presentation consistent with either PCOS or ovarian tumor.

Case A 7-year-old boy presents for evaluation of behavior problems in school that have been occurring for over 6 months. The boy's teacher recommended he be evaluated. The teacher reports that he tests at grade level, but he seems to make careless mistakes on schoolwork and has trouble maintaining attention to instruction. He does not finish his homework, and he often loses his homework, pencils, and books. The boy is often seen fidgeting at his desk. He blurts out answers and has difficulty waiting his turn. The parents tell you that they have seen similar traits at home, such as forgetting to do daily activities. For the past several years, he has been easily distracted. They view him as a happy, bright boy, and they report that he is very active. He has had normal vision and hearing screenings. The father self-reports that he was very similar as a child and still struggles with focus and concentration as an adult, but he has never received any help. The parents deny any major changes in the family situation. They would like medical help to improve their son's performance in school. The boy has been seen regularly for his well-child exams, has always met milestones, and has had normal exams. Today, he is quickly moving about the exam room. He looks at a book for a few moments, and then he looks out the window for a short time before interrupting his parents. Question What is the most appropriate intervention in this case? Advise the parents to hold the child back one year in school. Contact child protective services for suspected abuse. Initiate medication. Order head magnetic resonance imaging (MRI). Refer for counseling.

Correct answer: Initiate medication. Explanation This child meets criteria for attention deficit hyperactivity disorder (ADHD); it would be most appropriate to initiate medication at this time. If the diagnostic criteria are met with no concerning presentation for other disorders, treatment can begin immediately. The treatment of choice is prescription stimulant medications. ADHD is often seen (diagnosed or undiagnosed) in the family history. Often, young children may be less mature than their peers and could benefit from being held back a year in school, but the patient is testing at the appropriate grade level. If he repeats the year in school, this intervention will do nothing to address the root problem of ADHD. It is likely that he would continue having the same problems with inattentiveness and hyperactivity. A behavior change in a child could indicate some type of abuse or stressful situation for the child, but nothing in this child's history is indicative of child abuse. The behavioral problems have been present for longer than 6 months, they have been seen at home and at school, and nothing in his history or physical suggests abuse. Contacting child protective services is not appropriate at this time. ADHD is a clinical diagnosis. It is primarily based on patient history; neither imaging nor blood tests are routinely recommended. This boy's history and exam do not demonstrate any neurologic deficits suggestive of a brain tumor or other anatomic defect, so MRI is not recommended. A counseling referral would be reasonable if the child had a history suggestive of anxiety, depression, or psychosocial stressors, but counseling has very little to no role in the treatment of ADHD, especially in grade-school-aged children. Other forms of psychosocial intervention, such as parent training and classroom interventions, are recommended for children with ADHD.

Case A 48-year-old man presents to the family practice clinic for evaluation of fatigue, weakness, and nausea. He reports that his symptoms have progressively worsened over the last 6 - 8 months. He reports fatigue despite adequate sleep; he is also experiencing an overall feeling of muscle weakness, nausea with occasional vomiting, a weight loss of about 12 pounds, headaches, and muscle aches. His wife thinks he appears tanned year-round, despite a lack of sun exposure. He admits feeling anxious and somewhat irritable, but he denies any major psychosocial or traumatic events surrounding onset of symptoms. Prior to the onset of his symptoms, he was healthy and active. His past medical history reveals no chronic medical conditions, no medication use, and no history of surgery; he does not have any allergies. His family history is significant for thyroid disease in a sister and his mother; there is also a history of diabetes mellitus type I in a brother. He teaches high school, and he lives with his wife and children; he denies the use of tobacco, alcohol, and drugs. On physical exam, he is noted to be hypotensive and hyperpigmented. The remainder of his physical exam is normal. Several labs are performed, and the results are as follows: Urinalysis: Normal Complete blood count: Mildly decreased hemoglobin and hematocrit Comprehensive metabolic panel: Mildly decreased sodium and elevated potassium, rest normal Adrenocorticotropic hormone (ACTH): Elevated Cortisol (morning level): Decreased ACTH stimulation test: Decreased cortisol Question Once this patient is diagnosed, what choice represents the most important intervention for his presumed condition? Increase dietary sodium Initiation of antidepressant with anti-anxiety properties Initiation of steroids Refer to oncologist Restrict dietary potassium intake

Correct answer: Initiation of steroids Explanation This patient is presenting with a corticoadrenal insufficiency, which is also known as Addison's disease. In the United States, this rare disease is most often linked with autoimmune causes. The affected patient will have a variety of symptoms related to low glucocorticoid and mineralocorticoid production. Signs and symptoms include fatigue, nausea, vomiting, headaches, anorexia, myalgias, arthralgias, muscle weakness, anxiety, mental irritability, hyperpigmentation, hypotension, hyponatremia, hyperkalemia, anemia, and many more. The mainstay of treatment is initiation of steroids, often with oral hydrocortisone, unless urgent IV treatment is needed. Patients with Addison's disease commonly have hyponatremia and hyperkalemia. They can be instructed to increase dietary sodium, especially in times of exercise and hot weather. However, this alone will not address the underlying adrenal insufficiency, and failure to initiate steroid treatment will risk patient deterioration. With a variety of somatic and psychiatry complaints, a patient presenting like this may be falsely given a psychiatric diagnosis, and he may be treated with initiation of an antidepressant with anti-anxiety properties; however, the anxiety and irritability are symptoms of the adrenal disorder, and they should improve with treatment of the underlying disorder. It would be reasonable to monitor this patient and possibly treat him with antidepressants if his symptoms do not resolve. While Addison's disease is serious and chronic, it is not malignant. This patient's presentation with weight loss, nausea, and fatigue could be suggestive of cancer. However, with the laboratory values indicating adrenal insufficiency, it would be inappropriate to refer the patient to an oncologist. Even as a mild hyperkalemia is indicated on this patient's lab values, restriction of dietary potassium is not an appropriate treatment of his primary condition. The hyperkalemia is a result of the adrenal insufficiency. With appropriate treatment with steroids, such as hydrocortisone, the electrolyte abnormalities are likely to improve without additional intervention.

A 36-year-old G2P1 woman presents for her annual gynecological examination. In addition to routine testing, she is seeking advice on contraception. She currently takes medication as needed for migraine with aura. She has no other known medical conditions. She smokes half a pack of cigarettes daily. She is currently sexually active with her spouse and they remain undecided about their desire for more children in the future. Question What is the best contraceptive recommendation for this patient at this time? Combination oral contraceptives Intrauterine device Contraceptive vaginal ring Spousal vasectomy Bilateral tubal ligation

Correct answer: Intrauterine device Explanation Contraindications to estrogen-containing contraceptives include history of certain medical conditions, such as personal or family history of cardiovascular disease (deep vein thrombosis, pulmonary embolus, thrombotic events, myocardial infarction, cerebral vascular accidents), migraines with aura. Patients over 35 who smoke are also at increased the risk for embolic and thrombotic events. This patient should utilize either a progesterone-only or a non-hormonal contraceptive option. An intrauterine device is appropriate, as those currently available are either progesterone-only or non-hormonal. Both combination oral contraceptives and the contraceptive vaginal ring both contain estrogen as well as progesterone. Consequently, they would pose too great a risk for embolic and thrombotic events. Both spousal vasectomy and bilateral tubal ligation are permanent sterilization options, which would not be suitable for a patient undecided about future childbearing plans.

A 28-year-old woman presents 6 months after giving birth to her first child. She is interested in discussing birth control options. She was taking birth control pills when she got pregnant, but she had difficulty remembering to take the pills daily. She is married, monogamous, does not desire conception for the next several years, and wants a method that is independent of her management for its efficacy. Question Ico-delete Highlights What is the best contraception option to recommend for this patient? Vaginal ring Tubal ligation Intrauterine device Condoms Oral contraceptive pills

Correct answer: Intrauterine device Explanation Intrauterine device (IUD) is the correct response. The number out of every 100 women who experienced an unintended pregnancy within the first year of typical use of these contraceptive methods are as follows: Oral contraceptive pills (OCP) and the vaginal ring have a pregnancy rate of 9%. IUDs have a pregnancy rate of <1% (hormonal IUD/Levonorgestrel = 0.2%; copper IUD = 0.8%). Male condoms have a pregnancy rate of 18%. Tubal ligation pregnancy rates vary depending upon surgical approach (0.0% for bilateral salpingectomy; 0.5% for bilateral tubal ligation using rings/clips/cut and tie). What makes the IUD a better option than OCP in this case is the patient's previous history of not taking the pills regularly. While the vaginal ring is a better option than OCP, as it requires less maintenance, the patient "wants a method independent of her management," as she's afraid she will forget. Tubal ligation is not appropriate, as the patient does not desire permanent sterilization at this time. The IUD will eliminate the human error factor in birth control, and it can be effective for 3-10 years, depending on the type of IUD selected. It also is rapidly reversible if the patient changes her mind about having another baby before then.

A 4-year-old boy presents with poor weight gain, small size for his age, and dyspnea upon feeding. His mother notes that the child suffers from frequent upper respiratory tract infections. On physical exam, the child is underweight for his age. You note a precordial bulge, a prominent right ventricular cardiac impulse, and palpable pulmonary artery pulsations. You also find a widely split and fixed second heart sound as well as a mid-diastolic rumble at the left sternal border. Question What pharmacologic agent would be most appropriate in the medical management of this patient at this time? Lasix (furosemide) Coumadin (warfarin) Procardia (nifedipine) Inderal (propranolol) Indocin (indomethacin)

Correct answer: Lasix (furosemide) Explanation The correct response is Lasix (Furosemide). This patient's manifestations suggest a diagnosis of an atrial septal defect (ASD). ASD with moderate-to-large left-to-right shunts results in increased right ventricular stroke volume across the pulmonary outflow tract, creating a crescendo-decrescendo systolic ejection murmur. This murmur is heard in the second intercostal space at the upper left sternal border. Patients with large left-to-right shunts often have a rumbling mid-diastolic murmur at the lower left sternal border because of increased flow across the tricuspid valve. Definitive therapy for ASD includes closure of the defect, which is achieved surgically or through interventional catheterization. No specific or definitive medical therapy is available, but patients with significant volume overload or atrial arrhythmias may require specific drug therapy. For patients with large shunts and heart failure, diuretics, digoxin, and ACE inhibitors should be used before surgery. Although patients are at risk for paradoxical emboli formation, this patient does not present with manifestations consistent with vascular occlusion. For prophylaxis following the use of transcatheter occlusion devices, a common practice is to take aspirin for 6 months. Propranolol may be used along with morphine sulfate, phenylephrine, oxygen, volume expansion, and knee-chest or squatting positioning to manage Tet spells associated with Tetralogy of Fallot. Indomethacin is indicated in the closure of patent ductus arteriosus.

A 24-year-old woman with no significant past medical history presents for her yearly medical examination. She denies any medical problems. Her cardiac exam is notable for a mid-systolic click. This finding precedes a high-pitched, late systolic crescendo-decrescendo murmur, described as "whooping" or "honking," and was heard best at the apex and is without radiation. The provider noticed that these sounds occurred earlier in the cardiac cycle when the patient was standing and during the Valsalva maneuver as well as later when she squatted and during handgrip. Question What is the most likely diagnosis? Aortic regurgitation Aortic stenosis Mitral stenosis Mitral regurgitation Mitral valve prolapse

Correct answer: Mitral valve prolapse Explanation This patient's diagnosis is mitral valve prolapse (MVP). Most patients are asymptomatic, and it is more common in women and occurs most frequently between the ages of 15 and 30 years. The clinical course is most often benign. The most important finding is the mid or late (nonejection) systolic click. Systolic clicks may be multiple and may be followed by a high-pitched, late systolic crescendo-decrescendo murmur, which occasionally is "whooping" or "honking" and is heard best at the apex. The click and murmur occur earlier with standing, during the strain phase of the Valsalva maneuver, and with any intervention that decreases left ventricular (LV) volume, exaggerating the propensity of mitral leaflet prolapse. Squatting and isometric exercises (handgrip), which increase LV volume, diminish MVP; the click-murmur complex is delayed, moves away from S1, and may even disappear. Findings consistent with aortic regurgitation consist of tachycardia and widened pulse pressures. The classic auscultatory finding is a decrescendo, diastolic, high-pitched murmur loudest at the left sternal border that is accentuated with the patient leaning forward in full expiration. An Austin-Flint murmur may be evident, characterized by a low-pitched, mid-diastolic rumbling murmur due to blood jets from the murmur striking the anterior leaflet of the mitral valve, which results in premature closure of the mitral leaflets. Corrigan pulse ("water-hammer" pulse) is described as an abrupt distention and quick collapse on palpation of the peripheral arterial pulse; booming systolic and diastolic sounds auscultated over the femoral arteries represent Traube sign ("pistol-shot" pulse). The murmur of aortic stenosis is characteristically an ejection (mid) systolic murmur that commences shortly after the S1, increases in intensity to reach a peak toward the middle of ejection and ends just before aortic valve closure. It is characteristically low-pitched, rough, and rasping in character, and loudest at the base of the heart, most commonly in the second right intercostal space. It is transmitted upward along the carotid arteries. Exertional dyspnea, angina pectoris, and syncope are the three cardinal symptoms, which become most apparent beginning in the sixth decade. Rheumatic fever is the leading cause of mitral stenosis (MS). The first heart sound (S1) is usually accentuated and slightly delayed. The pulmonic component of the second heart sound (P2) also is often accentuated, and the two components of the second heart sound (S2) are closely split. The opening snap (OS) of the mitral valve is most readily audible in expiration at, or just medial to, the cardiac apex. It is followed by a low-pitched, rumbling, diastolic murmur, heard best at the apex with the patient in the left lateral recumbent position; it is accentuated by mild exercise carried out just before auscultation. Although usually asymptomatic in mild disease, fatigue, exertional dyspnea, and orthopnea are the most prominent complaints in patients with chronic severe mitral regurgitation (MR). On auscultation, S1 is generally absent, soft, or buried in the holosystolic murmur of chronic MR. In patients with severe MR, the aortic valve may close prematurely, resulting in wide but physiologic splitting of S2. A systolic murmur of at least grade III/VI intensity is the most characteristic auscultatory finding in chronic severe MR. It is usually holosystolic, is decrescendo, and ceases in mid-to-late systole in patients with acute severe MR. The systolic murmur of chronic MR is usually most prominent at the apex and radiates to the axilla. The systolic murmur of chronic MR not due to MVP is intensified by isometric exercise (handgrip) but is reduced during the strain phase of the Valsalva maneuver because of the associated decrease in LV preload.

A 25-year-old man presents with back pain and stiffness. He states he has had longstanding issues with back pain. He denies any trauma to his back. He has noticed associated increasing stiffness and general fatigue. He feels that these issues have gradually worsened over the last several months and are more persistent recently. He notes that the pain is much worse first thing in the morning, rating it a 6-7/10. Radiation occasionally occurs into the buttock areas and the patient feels the symptoms actually lessen with activity. Physical examination shows marked forward stooping of the thoracic and cervical spine with the lower spine showing the presence of a substantial reduction in lateral flexion. Question Ico-delete Highlights What would be the first-line pharmaceutical treatment to consider in this patient? TNF inhibitors NSAIDs Sulfasalazine Etanercept Corticosteroids

Correct answer: NSAIDs Explanation The correct response is NSAIDs. The patient history and physical examination described above is most likely a case of ankylosing spondylitis (AS). This is a chronic inflammatory disease that consists of many signs of symptoms, specifically significant back pain and progressive spinal stiffness. Many times, patients with AS will also have transient or persistent peripheral arthritis, as well as other manifestations such as anterior uveitis. Typically this is seen in male patients ages 20-30, although some may start having issues as early as their late teens. The goals of management of AS is to maximize the long-term health-related quality of life: relief of symptoms, maintenance of function, prevention of complications from spinal disease, and the minimization of extra-spinal and extra-articular manifestations and comorbidities. The first-line treatment for AS is nonsteroidal anti-inflammatory drugs (NSAIDs). Usually, empiric trials of different NSAIDs are warranted to gain satisfactory and beneficial relief of symptoms. TNF inhibitors and etanercept are both viable secondary options for patients who have failed relief from NSAID trials, but they are not considered first-line treatment. Sulfasalazine is sometimes used to help relieve symptoms in peripheral arthritic manifestations, but it is not useful for spinal or sacroiliac disease in AS. Corticosteroids have a very minimal impact on treating AS, and they could even worsen the osteopenia that is sometimes seen in this pathology.

A 44-year-old Caucasian man presents with chronic rhinitis, nasal congestion, and a decreased sense of smell. He has had these symptoms for several years, but this is the first time he has sought care for them. The patient thought he may have seasonal allergies and has intermittently tried numerous over-the-counter allergy treatments, some of which diminish symptoms temporarily. On physical exam, the patient's vitals are normal. His HEENT exam is positive for yellowish, boggy nasal mucosal masses protruding bilaterally in the nares. Surrounding tissue is relatively pale, without erythema. Some clear nasal discharge is noted. Posterior oropharynx is mildly inflamed without exudate. Ear exam is normal. Nodes within the neck and auricular and occipital regions are not enlarged. Heart, lung, and abdominal exams are normal, with the exception of a dry, nonproductive cough noted a few times. Question Ico-delete Highlights Based on this patient's history and physical, which of the following is the most likely diagnosis? Deviated septum Nasal foreign body Nasal polyps Nasopharyngeal carcinoma Rhinitis medicamentosa

Correct answer: Nasal polyps Explanation This patient most likely has nasal polyps, a consequence of long-term rhinosinusitis. Nasal polyps are benign mucosal masses that protrude into the nasal canal and may be unilateral or bilateral. They are associated with chronic allergies and a diminished sense of smell. A deviated nasal septum can produce decreased nasal airflow and present with some congestion-like symptoms, but the physical exam would reveal the septum lying off midline, without the mucosal masses characteristic of polyps. A nasal foreign body could present with a similar history of nasal congestion and rhinorrhea but is more frequent in young children at risk for putting objects into their noses. The physical exam would readily identify and distinguish a foreign body from nasal polyps. Neoplasms, such as nasopharyngeal carcinoma, are rare in comparison to the prevalence of nasal polyps and are typically asymptomatic until late in the disease process. A nasopharyngeal carcinoma is most common in patients of southern Chinese ethnicity. Symptoms might include rhinitis or nasal obstruction, similar to the presentation of this patient. However, a neoplasm may be more likely to include pain, nasal hemorrhage/bleeding, and unilateral symptoms if symptoms were present. Rhinitis medicamentosa is a condition resulting in severe rebound nasal congestion after prolonged and/or frequent use of nasal decongestants, such as phenylephrine. There are typically no distinguishing physical exam findings from this condition.

An 8-month-old boy presents for a routine evaluation. His mother states that the child is gaining weight and feeding appropriately and has been without fever, chills, dyspnea, or other abnormal objective signs. Upon physical examination, the examiner noticed a loud, harsh holosystolic murmur in the left third and fourth interspaces along the sternum that was associated with a systolic thrill. The vital signs were normal, with normal length and weight and a physiologic splitting of S2. There was no cyanosis, edema, or hepatomegaly. Question What is the most appropriate course of action for this patient? Cardiac catheterization Prescribe furosemide and captopril Observation Refer for cardiac MRI Immediate surgical repair

Correct answer: Observation Explanation Observation is the correct response. This patient's presentation represents a ventricular septal defect (VSD); the history and physical exam most likely suggests a small VSD. Children with small ventricular septal defects (VSDs) are asymptomatic and have an excellent long-term prognosis. Cardiac catheterization is used in circumstances of pulmonary hypertension of unknown reactivity or in assisting the decision making regarding the need for surgery by determining the pulmonary-to-systemic flow ratio in small-to-moderate defects with only mild left ventricular enlargement. Membranous and muscular septal defects may spontaneously close in childhood as the septum grows and hypertrophies; small shunts in asymptomatic patients do not require surgery or medical intervention. Uncontrolled CHF with growth failure and recurrent respiratory infection is an indication for surgical repair. In symptomatic patients, diuretics (furosemide) may be used to relieve pulmonary congestion. Angiotensin-converting enzyme (ACE) inhibitors reduce both the systemic and pulmonary pressures, thereby reducing the left-to-right shunt. Digoxin may be used if symptoms persist despite diuretic and ACE-inhibitor use. MRI is used only when ultrasonography is not feasible or when ultrasonographic findings are not diagnostic. It is also used as a substitute in cardiac catheterization for diagnostic purposes.

A 22-month-old girl presents with a 3-day history of decreased appetite. Her mother has also noticed a decrease in the child's sleeping pattern and an increase in irritability; she is favoring her right ear, and she had a fever of 102.4°F taken rectally the night before presentation. All of the aforementioned factors led to the girl's mother making an appointment with you. The patient does not have any significant medical history. Question What signs found during the physical examination would prompt a bacterial-induced diagnosis and necessitate treatment? Shiny, translucent tympanic membrane with increased mobility Opaque, erythematous tympanic membrane with decreased mobility Absence of a fluid line level behind the tympanic membrane External auditory canal with minimal amount of cerumen Tympanic membrane with the presence of a ventilating tube

Correct answer: Opaque, erythematous tympanic membrane with decreased mobility Explanation The correct response is opaque, erythematous tympanic membrane with decreased mobility. This is a classic example of a pediatric patient presenting with acute otitis media (AOM). A rapid onset of high fever, change of sleeping habits, alteration in appetite, and irritability are keys in regards to the child's medical history. She also is most likely having otalgia due to the fact that is she is favoring that ear. Both the American Academy of Pediatrics and the American Academy of Family Physicians define 3 criteria that must be present in order to make the diagnosis of AOM: acute onset of symptoms the presence of fluid in the middle ear (evidenced by the presence of a fluid line or visibility of purulent fluid) signs and symptoms of middle-ear inflammation (e.g., erythema, bulging, decreased mobility via pneumatic insufflation) Any otoscopic examination revealing a shiny, translucent membrane with no fluid line present and the evidence of tympanic membrane mobility would be considered a normal finding. Cerumen is also commonly found in the external portion of the ear, but not in the middle ear, and it is also normal for it to be visible in various amounts in the canal. Ventilation tubes, sometimes called ear tubes, tympanostomy tubes, or myringotomy tubes, are literally cylinders placed in the tympanic membrane to allow air into the middle ear. Ventilation tubes are usually recommended for patients who experience repeated AOM, have hearing loss due to otitis media with effusion, a malformation of the tympanic membrane or Eustachian tube, or barotrauma. More commonly, tubes are necessary in the pediatric population due to the common occurrence of AOM, but this need can be seen in adolescents or even adults.

An 82-year-old man presents for evaluation of an itchy hive-like rash on his abdomen. He has had it for months. Over the last few days, the patient reports the character of the rash has changed; it now resembles blisters. Other than some skin irritation, he feels healthy. He has tried multiple over-the-counter topical treatments, but they have been unsuccessful in treating the rash. He denies any changes in soaps, lotions, laundry detergents, or anything else that may have come in contact with his skin. He also denies unusual travel, pets, or hobbies. No close contacts have reported a similar condition. He takes no medications, and he does not have any chronic illnesses. On physical exam, multiple clusters of bullae are noted across the trunk bilaterally, with some distribution on both anterior and posterior surfaces. There are still some remaining pink-red lesions; they are scattered among the bullae. The bullae are 1-3 cm in size; they are tense and do not easily rupture. They do not extend into normal skin with pressure. Pressure on the normal skin does not produce a blister. The remainder of his physical exam is normal. Question Once you confirm the diagnosis, what intervention is most appropriate for this patient's condition? Apply trichloroacetic acid (TCA) or bichloroacetic acid (BCA). Cryotherapy of bullous lesions. Dermabrasion Oral prednisone Topical acyclovir

Correct answer: Oral prednisone Explanation This patient is presenting with bullous pemphigoid (BP), a benign autoimmune skin disorder characterized by the presence of bullae (or blisters). BP is relatively rare and tends to affect the elderly population. The preceding rash is typically pruritic and appears as urticarial plaques, possibly resembling erythema multiforme. This stage may last for months, and then the skin blisters. When the blisters rupture (typically within 1 week), the skin heals. Unless the BP is extensive and generalized, the prognosis is good. Oral steroids, such as prednisone, have been the primary treatment, but other approaches, such as antihistamines for pruritus, topical steroids, antibiotics, methotrexate, and azathioprine, may all have a role in treating BP. The use of trichloroacetic acid (TCA) or bichloroacetic acid (BCA) is most appropriate for condyloma acuminata (genital warts). These acids are caustic agents that destroy tissue. They are not indicated in the treatment of BP. Cryotherapy of bullous lesions is not recommended, especially as the bullae tend to resolve spontaneously in approximately 1 week, with normal healing expected. Cryotherapy is not appropriate. Dermabrasion is a popular skin procedure for aging skin. It consists of mechanical removal of surface skin, producing a similar effect to chemical peels. Neither dermabrasion nor chemical peel is a good treatment for BP. Topical acyclovir is an antiviral, which is useful in the treatment of herpes conditions. Herpes family infections could be on the differential, especially with a blister-like appearance, but herpes vesicles are typically much smaller, and none of the herpes infections fit the presentation and distribution of this patient's "rash." BP is an autoimmune condition, and an antiviral medication would not be appropriate.

Case A 4-year-old boy begins to develop hematomas. He mentions knee pain to his mother, and she notices his knees are swollen and painful. His mother is very concerned because she knows her father had a bleeding disorder. Question What laboratory test is most likely to be abnormal in this patient? Fibrin split products Platelet count PT PTT Thrombin time

Correct answer: PTT Explanation The boy has hemophilia A. The partial thromboplastin time (PTT) is prolonged with hemophilia A. The activated partial thromboplastin time (aPTT or PTT) is a measure of the intrinsic coagulation pathways and will be abnormal in hemophilia A. The fibrin split count would not be elevated with hemophilia A. The platelet count would be in the normal range with hemophilia A. The prothrombin time (PT) is normal with hemophilia A. The prothrombin time (PT) is a measure of the extrinsic pathway and thus would be normal in this case. The thrombin time would not be abnormal with hemophilia A.

A newborn child is routinely evaluated in the pediatrician's office 1 month after delivery. The mother reports that the patient is behaving normally and is feeding well. The physical exam is remarkable for a murmur, which is located at the second left intercostal space. The murmur is continuous throughout cardiac systole and diastole, non-radiating, and of a "machinery" quality. There is additionally a widened pulse pressure. The skin and mucosa are without cyanosis, and there is no evidence of fluid retention. Question What is the most likely diagnosis? Ventricular septal defect Atrial septal defect Transposition of the great arteries Tetralogy of Fallot Patent ductus arteriosus

Correct answer: Patent ductus arteriosus Explanation This infant is most likely exhibiting signs consistent with patent ductus arteriosus. Patent ductus occurs more commonly in female patients and has an incidence rate of up to 50% in infants born with a birth weight of less than 1 kilogram. Patent ductus arteriosus is characterized by an abnormal patency of the ductus arteriosus, which delivers blood from the right side of the heart to the systemic circulation during fetal life. Blood typically shunts from the higher-pressured left side (systemic circulation) to the lower-pressured right side (the pulmonary circulation). Normally, this ductus closes following spontaneous respiration of the newborn, as well as a lowering of endogenous prostaglandin and adenosine levels. The patency is normally sealed with fibrous tissue within 2-3 weeks. With a ventricular septal defect, blood flows from the left ventricle to the right ventricle and presents as a harsh, blowing holosystolic murmur with a thrill localized to the fourth left intercostal space. This murmur may decrease with Valsalva and handgrip. Atrial septal defect is characterized by a shunting of blood from the left atrium to the right atrium, whose murmur is mild in intensity, located in the pulmonic area, and is associated with a widely-split S2 heart sound. Transposition of the great arteries is the most common cyanotic congenital heart lesion that presents in neonates. It is a severe defect with failure of rotation of the great vessels, leaving the aorta to arise from the right ventricle and the pulmonary artery from the left ventricle. Characteristics include intense generalized cyanosis present at birth, lack of a murmur unless associated with ventricular septal defect, rapid and progressive congestive heart failure, and increased intensity of the second heart sound. Tetralogy of Fallot is the most common cyanotic congenital heart defect, but it often does not present in the neonatal period. It can present later in childhood. Transposition of great arteries is the cyanotic heart defect presenting most often in the neonatal period, as all transposition patients are cyanotic from birth. The ventricular septal defect is among the most common lesions in the VACTERL association.

Case A 72-year-old man is evaluated at his bedside following hospital admission for a 1-year history of progressive dyspnea, nonproductive cough, weight loss, low-grade fevers, fatigue, and myalgias. His past medical history is remarkable for atrial fibrillation, for which he takes amiodarone, hypercholesterolemia, and recurrent urinary tract infections, for which his urologist prescribed nitrofurantoin on a chronic, prophylactic basis. He denies any cigarette use, history of murmurs or coronary artery disease, chills, fatigue, rhinitis, otalgia, chest pain, wheezing, hemoptysis, syncope, abdominal pain, rashes, peripheral edema, diaphoresis, arthralgias, vomiting, or urinary complaints. A bedside echocardiogram and electrocardiograms are unremarkable for abnormalities; a chest X-ray revealed peripheral reticular opacities at the lung bases and a generalized honeycombing pattern (refer to image). Question What is the next best step in the diagnosis of this patient? Perform a high-resolution computed tomography (HRCT) Surgical lung biopsy by video-assisted thoracoscopic surgery (VATS) Order C-reactive protein levels and an erythrocyte sedimentation rate Order polymerase chain reaction (PCR) testing for common respiratory viruses Perform an upper endoscopy to assess for the presence of esophageal reflux

Correct answer: Perform a high-resolution computed tomography (HRCT) Explanation This patient's most likely diagnosis is idiopathic pulmonary fibrosis (IPF). It is defined as a specific form of chronic, progressive fibrosing interstitial pneumonia of unknown cause, primarily occurring in older adults, limited to the lungs, and associated with the histopathologic and/or radiologic pattern of usual interstitial pneumonia (UIP). The chest radiograph lacks diagnostic specificity for idiopathic pulmonary fibrosis. Virtually all patients with IPF have an abnormal chest radiograph at the time of diagnosis. The typical findings are peripheral reticular opacities (netlike linear and curvilinear densities) predominantly at the lung bases. Honeycombing (coarse reticular pattern) and lower lobe volume loss can also be seen. High-resolution computed tomography (HRCT) findings are significantly more sensitive and specific for the IPF diagnosis and are essential in the diagnostic pathway of IPF. On HRCT images, idiopathic pulmonary fibrosis is characterized by patchy, predominantly peripheral, predominantly subpleural, and bibasilar reticular opacities. Surgical lung biopsy specimens can be obtained via open lung biopsy or video-assisted thoracoscopic surgery (VATS). It is not essential for diagnosis, however, due to the high-quality evidence regarding HRCT specificity for the recognition of histopathologic UIP pattern. The C-reactive protein value and erythrocyte sedimentation rate can be elevated in patients with idiopathic pulmonary fibrosis; however, this finding is nondiagnostic. Although respiratory viruses have been considered a particularly likely cause of acute exacerbations of IPF (AE-IPF) due to similarities in clinical and radiologic presentation between and AE-IPF and viral pneumonitis, polymerase chain reaction (PCR) testing reveals common respiratory viral infection in only a minority of patients with AE-IPF. Viral infection is not detected in most cases of AE-IPF. There is a high prevalence of gastroesophageal reflux (GER) in patients with IPF. Although the use of anti-reflux medications is associated with a longer survival time in patients with IPF, an upper endoscopy is not indicated in the evaluation of IPF.

A 21-year-old woman presents with a severe headache; it is accompanied by sweating and palpitations. She was seen by her primary care physician earlier in the day who admitted her from the office when her blood pressure was recorded several times at 210/98 mm Hg. The patient has no known medical or surgical history, and she is currently on a daily multivitamin. She states that she has been having her symptoms episodically over the last 2 weeks, but this is the first time she has been seen by a physician. The patient is found to have elevated urinary and plasma metanephrines and catecholamines. Question What antihypertensive agent should be used for this patient? Lisinopril Phenoxybenzamine Metoprolol Lasix Hypertension should not be treated at this time.

Correct answer: Phenoxybenzamine Explanation The correct answer is phenoxybenzamine. The patient's young age suggests that she is suffering from a form of secondary hypertension. Given her paroxysmal headaches, diaphoresis, and palpitations accompanied by hypertension, she is likely suffering from a pheochromocytoma. Pheochromocytomas are benign tumors that arise from the adrenal glands. The tumors secrete norepinephrine, which leads to hypertension, tachycardia, headache, and diaphoresis. Diagnosis is made based on elevation in urinary and plasma metanephrines and catecholamines, followed by tumor location on CT or MRI. The patient requires removal of the pheochromocytoma, but preoperative control of her blood pressure is necessary. Alpha blockers, such as phenoxybenzamine or prazosin, are the preferred antihypertensive agents in pheochromocytoma. Lisinopril is an ACE inhibitor and would not be considered first-line treatment in the control of hypertension in pheochromocytoma. Metoprolol is a beta blocker. Beta blockers may only be used in the treatment of pheochromocytoma after the patient has received an alpha blocker. If a patient with pheochromocytoma receives a beta blocker as first-line treatment, the patient will develop unopposed alpha agonism, worsening their hypertension. Lasix is a diuretic and is not recommended as first-line treatment in the control of hypertension in pheochromocytoma. The patient's blood pressure is severely elevated and she requires control of her blood pressure prior to the removal of her pheochromocytoma.

Case A 22-year-old man presents with sudden onset of shortness of breath and right-sided chest pain. Symptoms began abruptly yesterday. He felt well prior to the onset of symptoms. He denies fever, hemoptysis, and upper respiratory symptoms. He smokes one pack per day; he has an otherwise non-contributory past medical history. On physical exam, the patient is in mild respiratory distress. He has a slightly elevated heart rate and respiratory rate. He is normotensive. His trachea appears deviated to the left. On pulmonary exam, breath sounds are diminished on the right. Hyperresonance is noted on percussion of the right chest compared to the left. Other than tachycardia, his cardiovascular exam is normal. Question What test finding is most diagnostic for your suspected diagnosis of this patient? Blunting of costophrenic angles on chest X-ray (CXR) Increased pH on arterial blood gas (ABG) Oxygen saturation less than 90% on pulse oximetry Pleural line on chest X-ray (CXR) Sputum smear positive for acid-fast bacilli (AFB)

Correct answer: Pleural line on chest X-ray (CXR) Explanation This patient is presenting with a spontaneous primary pneumothorax. A finding of a pleural line on chest x-ray (CXR) is diagnostic for this condition. A pneumothorax is a condition in which air is introduced into the lung cavity, either spontaneously or by trauma. In young people without known pulmonary pathology (especially more common in smokers), a pneumothorax may occur spontaneously. In older patients, other causes, such as a lung tumor or severe pulmonary disease, can lead to the collapse of the lung. As the lung collapses, the line from the edge of the pleura may be visible on CXR. Blunting of costophrenic angles on CXR would indicate some type of pleural effusion or fluid in the lungs. This patient's history and physical are not consistent with a pleural effusion (other than some dyspnea may be present). Specifically, percussion over a pleural effusion should produce a dullness, not a hyperresonance. Increased pH on ABG can certainly be seen in cases of respiratory alkalosis, associated with a pneumothorax. However, increases in pH can occur with numerous other conditions; therefore, an increase is not diagnostic for tension pneumothorax. Oxygen saturation may drop below 90% in cases of a pneumothorax; conversely, the oxygen saturation may be normal in milder cases of pneumothorax. However, as with the ABG, decreased oxygen saturation is a common finding with various pulmonary conditions; it alone is not diagnostic for pneumothorax. A sputum smear positive for acid-fast bacilli indicates a Mycobacterium pulmonary infection, such as tuberculosis. Classically, tuberculosis does not produce tracheal shift or hyperresonance on exam. This patient's history is not consistent with a pulmonary infection either.

A 42-year-old morbidly obese woman is referred back to her primary care provider to supervise her weight loss program. A surgeon evaluated her for a "gallbladder attack" 1 month ago, which then resolved. She currently denies abdominal pain. On ultrasound, there are visible stones in the gallbladder. She declined to have surgery until she achieves her initial weight loss goal of 50 pounds. She is on a supervised healthy diet of about 1200 kilocalories daily. She exercises 1 hour a day 6 days per week. She is losing about 15-16 pounds a month. Question Ico-delete Highlights What intervention would be most effective at preventing the recurrence of her gallbladder "attack" until she achieves her weight loss goal? Prescribe orlistat Prescribe ursodiol Recommend over-the-counter omeprazole Reduce patient's exercise Restrict caloric consumption further

Correct answer: Prescribe ursodiol Explanation Both morbid obesity and rapid weight loss are risk factors for the development of cholecystitis. Gallstones may be present in the gallbladder and remain asymptomatic, or the gallbladder walls may become inflamed, resulting in cholecystitis. Ursodiol is approved for prevention of gallstones in obese patients with rapid weight loss. It would be the best choice for health maintenance and delaying or preventing the need for surgical cholecystectomy. Orlistat is a prescription medication approved for treatment of obesity. Its mechanism is to block fat absorption. It does not have a direct role in health maintenance or prevention of cholecystitis or cholelithiasis. In fact, it has pronounced gastrointestinal side effects. Over-the-counter omeprazole is a commonly used proton-pump inhibitor (PPI); it is helpful in lowering acid in the stomach and treatment of gastroesophageal reflux disease. It has not been shown to help gallbladder disease, and there is some evidence that PPIs may worsen it. Reducing the patient's exercise may aggravate her condition, negatively impacting her weight loss. Exercise has an inverse relationship with cholecystitis and rates of cholecystectomies, so this patient should be encouraged to continue frequent exercise. This patient should not be instructed to restrict caloric consumption further. She is already losing weight quite rapidly at around 4 pounds per week. Her daily caloric intake is low; recommending further restriction will likely discourage her, and there will not be any benefit in regard to her gallbladder disease.

Case A 34-year-old man presents due to something being "wrong" with his left ear. He reports his hearing has been gradually declining, but most recently he noticed some discomfort and malodorous discharge draining from this ear. He denies any trauma to the ear and any symptoms in his right ear. Upon further questioning, he does admit some tinnitus and mild vertigo. He otherwise feels well. He denies nasal symptoms, headache, sore throat, and fever. His past medical history is unremarkable; he has no known medical conditions or history of surgeries; he takes no medications and has no allergies. He lives with his wife and 2 children; he works as an office manager, and he denies the use of alcohol, tobacco, and drugs. On physical exam, his vitals are normal. Examination of the left ear reveals mucopurulent drainage within the external auditory canal. The tympanic membrane is disrupted by a retraction pocket within the upper portion, with thick yellow debris and a polyp protruding from the pocket. Hearing tests are not performed. The right ear reveals mild tympanosclerosis on the tympanic membrane, but it is otherwise normal. Hearing tests are not performed. The remainder of the patient's exam is normal. Question What intervention is most appropriate in this case? Initiate topical antibiotics. Initiate topical imiquimod or podophyllin. Recommend ear plugs when swimming but no active treatment. Refer for radiation. Refer for surgery.

Correct answer: Refer for surgery. Explanation This patient is presenting with a cholesteatoma, which is a benign neoplasm of the tympanic membrane. It is considered a complication of chronic otitis media. The cholesteatoma is an epidermal inclusion cyst. Complications can include infection, and more significantly, erosion into bone and nerve damage. The treatment of choice is a referral to surgery. This condition could be potentially mistaken for otitis externa, for which it may be appropriate to initiate topical antibiotics. However, topical antibiotics will not help with cyst resolution, and they will not prevent the more serious complications of a cholesteatoma. If there was a secondary bacterial infection in the cholesteatoma cyst, topical antibiotics could be justified, but only in addition to surgical referral. If the growth in the patient's ear was a wart, one may consider initiation of topical imiquimod or podophyllin. However, warts can usually be distinguished by visual exam, and they would be highly unlikely to affect the tympanic membrane. These medications (and really, all medications) have no role in the treatment of cholesteatoma. Again, if the provider mistakenly diagnoses otitis externa as the cause of this patient's ear discharge, they may recommend earplugs when swimming in order to prevent recurrence. This can be a common mistake of providing treatments by phone based on the patient's report only and no physical exam. To provide no active treatment for the cholesteatoma is inappropriate. A referral for radiation is not recommended. Typically, excision is adequate treatment. A cholesteatoma is considered a benign neoplasm and does not metastasize.

A 27-year-old man is admitted to the hospital following a motor vehicle accident. He sustained lacerations to his arms bilaterally and has fractures of the right tibia and fibula. A cast is placed and the patient is scheduled for surgery the following day. A few hours after the cast is placed, he develops severe pain; the pain is unresponsive to several doses of intravenous morphine. His pain increases when he extends his right leg. Peripheral pulses are weak but present. Question Ico-delete Highlights Based on the history and physical, what should be done next? Take X-ray of the tibia and fibula. Perform MRI of the right leg. Remove cast and check compartment pressure. Perform ultrasound of the lower extremity. Monitor and continue giving narcotics.

Correct answer: Remove cast and check compartment pressure. Explanation The patient should have the cast removed and compartment pressure measured. This patient is showing signs of compartment syndrome. Compartment syndrome develops when there is an accumulation of pressure within a muscle compartment. It typically occurs following trauma to a limb. Patients typically present with pain unrelieved by analgesia as well as pain with extension of the involved limb. As the swelling within the compartment worsens, patients develop absence of pulse, paresthesias, pallor, and poikilothermia. The compartment pressure should be checked; if it is elevated, the patient should be taken for fasciotomy. An X-ray of the tibia and fibula would not demonstrate compartment syndrome. An MRI of the right leg would not be useful in this patient. An ultrasound of the lower extremity would be useful in diagnosing a lower extremity DVT. However, the patient's presentation is highly suggestive of compartment syndrome, and it should be ruled out before pursuing any other diagnoses. Monitor and continue giving narcotics is incorrect. If compartment syndrome is left untreated, it can lead to tissue death.

Case A 34-year-old pregnant woman presents with what she describes as "tingling in her right arm"; she is at her 34th week of gestation. This is her 2nd pregnancy. It comes on along her palm, and it has been increasing in severity. She also notices it more at night and while attempting to carry her shopping bags. Of late, her pain has been so bad that she has been losing sleep. Examination reveals a positive Phalen's test; there is no weakness or atrophy of the thenar muscles. She is not a known diabetic or hypertensive, and she is otherwise healthy. Her thyroid function is normal. Her primary care physician advised treating with NSAIDs and vitamin supplements, neither of which have worked after 3 months of regular use. Question What is the next best step in management? Surgical release Oral corticosteroids Splint in flexion Rest and neutral splinting Corticosteroid injection

Correct answer: Rest and neutral splinting Explanation Rest and neutral splinting is the correct answer. Pregnancy predisposes women to carpal tunnel syndrome, especially during the 3rd trimester, probably due to changes in fluid composition and balance. Resolution is typically seen following delivery. Rest and placing the hand in a neutral splint are sufficient in the majority of patients, and doing so affords relief until delivery. Surgical release is incorrect. In the setting of pregnancy, carpal tunnel syndrome is reversible, making surgical release unnecessary. Furthermore, surgery is not indicated in any patient with carpal tunnel syndrome without a trial of conservative treatment, except when there is evidence of severe nerve entrapment, such as muscle atrophy, demonstrable muscle weakness, and/or nerve conduction studies showing impaired conduction. Oral corticosteroids are incorrect. Rest is sufficient, and corticosteroids are to be avoided in pregnancy except in very specific conditions for the benefit of the fetus. Splinting in flexion is incorrect. Splinting is done in a neutral wrist position. Flexion exacerbates carpal tunnel symptoms Corticosteroid injection is incorrect. It is a possible option if rest and splinting in neutral position fails.

A 2-year-old girl has a 2-week history of small wounds on the sides of her mouth. On examination, you note she has pale conjunctivae, a magenta tongue, and macerated lips (in addition to the angular stomatitis). Her mother notes that she is a very fussy eater; she is underweight and was diagnosed in infancy with a malabsorptive syndrome. Question Ico-delete Highlights The child is most likely deficient in what nutrient? Thiamine Riboflavin Vitamin K Vitamin D Iron

Correct answer: Riboflavin Explanation In cases of riboflavin (or vitamin B2) deficiency, patients present with angular stomatitis and cheilosis. On examination, they are pale, have atrophic glossitis, and the tongue may appear magenta. They may also have a sebaceous dermatosis with greasy material in their nasolabial folds, alae nasi, and genitals. Causes include inadequate dietary intake and malabsorption. Dietary sources of riboflavin include milk, cheese, meat, green vegetables, and enriched cereal products. Vitamin B1 (or thiamine) deficiency results in beriberi, which is characterized by: a bilateral symmetric peripheral neuropathy beginning in the legs Wernicke-Korsakoff syndrome, which is comprised of nystagmus, ophthalmoplegia, ataxia, memory loss, and confabulation congestive heart failure with tachycardia, peripheral edema, and cardiomegaly In cases of vitamin K deficiency, patients present with bleeding tendencies, such as epistaxis, menorrhagia, and hematuria. The prothrombin time (PT) and the activated partial thromboplastin time (aPTT) are usually prolonged. In cases of vitamin D deficiency, children can present with an inability to walk unsupported due to muscle weakness and lower limb skeletal deformities, such as genu varum and genu valgum. They may also have prominent costochondral junctions. In cases of iron deficiency, patients can present with feeling weak, dizzy, and tired; they may experience syncope. On examination, they have pale conjunctivae and koilonychia (spooning of the nails).

A 47-year-old man presents for his annual physical exam. His past medical history is not significant, and he is not currently on any medications. He consumes 2 beers weekly and does not smoke. His blood pressure is normal during this visit. His primary care physician orders a fasting lipid panel with the following results: Component Value Triglyceride 135 HDL 50 LDL 220 Question Ico-delete Highlights In addition to diet and exercise, what medication should be started on this patient? Cholestyramine Fenofibrate Nicotinic acid Simvastatin Ezetimibe

Correct answer: Simvastatin Explanation The correct answer is simvastatin. This patient has elevated LDL, decreased HDL, and a normal triglyceride level. Simvastatin, an HMG-CoA reductase inhibitor, will decrease LDL and increase HDL levels. The patient has no history of liver disease, which is the main contraindication for using HMG-CoA reductase inhibitors. Cholestyramine is a bile acid sequestrant that may be used in combination with HMG-CoA reductase inhibitors, but it is not recommended for monotherapy in patients with elevated LDL levels. Fenofibrate, a fibric acid, is primarily used for the treatment of hypertriglyceridemia. The above patient has a normal triglyceride level. Nicotinic acid (niacin) may be used in combination with HMG-CoA reductase inhibitors, but it is not typically used in monotherapy for elevated LDL levels. Ezetimibe acts by blocking cholesterol absorption in the intestines. It is currently recommended for those who fail to respond to HMG-CoA reductase inhibitor therapy. This patient has newly diagnosed hyperlipidemia, so ezetimibe would not be an appropriate first-line treatment.

A 28-year-old man presents with a rash. The lesions, which are mildly pruritic, are located on his arms and legs. They have been present for about 3 days without change or resolution. He has tried over-the-counter anti-itch creams, but they have been ineffective. The patient reports that he was seen approximately 1 week ago for some blister-like lesions on his penis. He was given an antiviral medicine, and those lesions resolved. He is wondering if he was misdiagnosed and if the two rashes are related. He admits to feeling some malaise over the last 2 weeks, but he is otherwise healthy. He denies fevers, unusual travel, medication use (except for as listed above), and known allergies. He has no known chronic conditions. On physical exam, vitals are normal; the patient is in no apparent distress. A pink-to-red papular rash is observed on the backs of the hands and feet and extensor surfaces of the arms and legs. The individual lesions are quite distinct; they have a red center, and they are surrounded by a pale ring and then another outer ring of red, inflamed tissue. The remainder of his physical exam is normal. Question Ico-delete Highlights What test will confirm the suspected diagnosis? Complete blood count (CBC) with differential Fungal culture Herpes culture Heterophile antibody Skin biopsy

Correct answer: Skin biopsy Explanation This patient is presenting with a target lesion rash; it is typical of erythema multiforme, which is a relatively common type IV hypersensitivity reaction. The confirmatory test is skin biopsy. In about 50% of cases of erythema multiforme, no cause is identified. Other causes are infection and medications. When a cause is identified, recent herpes simplex virus (HSV) infection is the most common. In this patient's case, he likely was appropriately diagnosed and treated for his HSV infection, then developed the erythema multiforme rash. Erythema multiforme is an acute self-limited condition and can range in severity from a rash and mild malaise and pruritus (erythema multiforme minor) to a much more severe condition, involving mucosal surfaces and possible desquamation with erythema multiforme major. A complete blood count (CBC) with differential is a useful test in many conditions in which infection is on the differential, but it is rarely confirmatory for dermatologic conditions. This patient may have a slight leukocytosis, but the differential may be variable, with possible monocytosis and/or neutrophilia. A fungal culture would not be useful for the diagnosis. Ringworm (tinea circinata) may come to mind with ring-shaped lesions, but these fungal lesions do not have the characteristic red center with target appearance. Ringworm lesions more typically feature central clearing with a red outer ring. A herpes culture would have been useful in confirming the diagnosis of the initial rash, which the patient described as blister-like on his penis. Culturing the current erythema multiforme target lesions would not yield HSV. A heterophile antibody is a test for infectious mononucleosis. A mild rash may occur with mononucleosis, but there are typically several other key symptoms and findings, such as pharyngitis, lymphadenopathy, and profound fatigue.

You are reviewing incoming labs for another physician while they are out of town. You see the following laboratory results for a 51-year-old man. The electronic health record also shows the labs that he had drawn 4 months ago, allowing you to compare results. Current values 4 months ago Normal Units Lipid Panel Cholesterol 224 288 ≤200 mg/dL Triglyceride 190 224 ≤150 mg/dL HDL 42 38 40-59 mg/dL LDL 144 206 ≤100 mg/dL VLDL 38 44 12936 mg/dL Complete Metabolic Panel (CMP) Sodium 133 138 134-144 mmol/L Potassium 3.7 3.6 3.4-4.9 mmol/L Chloride 100 102 100-109 mmol/L HCO3 20 25 20-31 mmol/L Glucose 94 98 70-99 mg/dL Bun 15 16 7-18 mg/dL Creatinine 1.1 0.9 0.6-1.2 mg/dL Calcium 9.2 9.1 8.8-10.5 mg/dL Albumin 4.1 3.9 3.5-5.0 g/dL Total Protein 7.7 7.4 6.4-8.2 g/dL AST (SGOT) 37 30 15-37 U/L ALT (SGPT) 40 38 5-43 U/L Alk Phosphatase 83 89 50-136 U/L Total Bilirubin 0.6 0.8 0.1-1.2 mg/dL eGFR >60 >60 >60 mL/min/1.73m2 Question Interpret the laboratory values and changes. What is the most likely change this patient made to explain the change in test results? Began a low-sodium diet Began exercising regularly Started an HMG CoA reductase inhibitor Started an omega 3 fatty acid supplement Stopped smoking

Correct answer: Started an HMG CoA reductase inhibitor Explanation From the dramatic decreases in this patient's total cholesterol (23%) and low-density lipoprotein (LDL) (30%), the most likely explanation is that the patient started an HMG CoA reductase inhibitor (statin). A statin medication would be indicated based on this patient's initial laboratory data 4 months ago (assuming his clinical history did not present contraindications). Statin medications can be expected to achieve a reduction in LDL of 18-55%, increase of high-density lipoproteins (HDL) of 5-15%, and a reduction in triglycerides of 7-30%. Many lifestyle changes are recommended for reducing cardiovascular risk. Healthcare practitioners should not discount the benefit of a healthy diet, smoking cessation, and increased physical activity, but they should be aware of the limitations of quantifying the benefits of such changes. Patients may be easily discouraged if their changes do not reflect an improvement in their lab parameters. A low-sodium diet has been recommended primarily for patients with hypertension, not hyperlipidemia. A recent meta-analysis has been able to quantify approximately a 2.5% increase in cholesterol and 7% increase in triglycerides in patients placed on reduced sodium diets. The benefit to blood pressure is minimal. Because this patient's cholesterol dropped, a low-sodium diet would not account for his lab changes. The slight decrease in the patient's serum sodium relates more to his fluid status and not to his dietary intake of sodium. It is widely accepted that exercise is beneficial for overall health, as well as cardiovascular health. But when attempting to quantify the benefit of exercise on a standard lipid panel, small differences are seen; primarily, the differences are a small increase in HDL and a small reduction in triglycerides. No significant difference is detected in overall LDL measurements in studies evaluating the effects of exercise. Some researchers have observed improvements in the particle size and subfractions of LDL, but these benefits are not measurable on a standard lipid panel. Several studies have shown reductions in cardiovascular events and mortality in patients treated with omega 3 fatty acid supplements, such as fish oil, but omega 3 fatty acids do not reduce LDL significantly. Their primary quantifiable result is decreased triglycerides and increased HDL. They also have anti-inflammatory properties. Smoking cessation carries a multitude of benefits for the health of a smoker, but the expected benefit to their lipid profile is limited. Studies have demonstrated a slight increase in HDL, but no significant reduction of LDL. This patient's reduction in LDL cannot be attributed to smoking cessation.

A 49-year-old Caucasian man is well known to your practice; he presents with pain in his left lower extremity. Upon further questioning, the patient describes doing construction work 3 months prior when he jumped onto his feet from a height of about 5 feet. Since this episode, he has noted issues of increasing left sided hip and knee pain. He describes the pain as radiating into the left groin and front middle thigh area. The pain is relieved with sitting, and it is aggravated by walking and climbing up stairs. The patient denies any paresthesias, numbness, bowel/bladder dysfunction, fever, night sweats, or chills. Radiographic interpretation includes the presence of a crescent sign as well as marked irregularity of the left femoral head with sclerosis. Question Ico-delete Highlights Given these findings, what would you expect to see on a radiographic image taken of the left hip? Subchondral collapse Lytic lesions Femoral head fracture Bone cyst Bone tumor

Correct answer: Subchondral collapse Explanation The correct response is subchondral collapse. Avascular necrosis, sometimes also referred to as osteonecrosis, is osseous cell death resulting from vascular compromise. Common sites that are affected include the proximal or distal femoral head or even the ankle, shoulder, or elbow. Common causes include but are not limited to corticosteroid use, alcoholism, trauma, systemic lupus erythematosus, pancreatitis, gout, or even sickle cell disease. Radiographic findings will include mild density changes early on, sclerosis and a more progressed disease state will lead to the pathognomonic crescent sign. When repair begins at the interface between necrotic and viable bone, this produces a sclerotic margin. Eventual mechanical failure of trabecular bone at this interface results in progressive microfracture and collapse of the adjacent dead subchondral trabeculae and this leads to the subchondral radiolucent area along the fracture line, which is the crescent sign. Later disease will show joint-space narrowing and degenerative changes in the acetabulum. Lytic lesions are most commonly seen in patients with multiple myeloma. A fractured femoral head may be seen as the result of a high-impact trauma involving significant force, falls in older patients, or various illnesses or disease states that significantly affect bone integrity, such as vitamin D deficiency, systemic lupus erythematosus, or cancer. Both of these potential answers are not as likely. A bone cyst is described as a fluid-filled hole that develops inside a bone. These most commonly occur in children and young adults and very rarely cause significant symptoms. The cause of bone cysts is unclear; they will not usually cause serious health issues. This does not match this case. A bone tumor is an area of abnormal growth of cells within the bone and may either be malignant or benign. Causes of bone tumors are generally unknown but are seen commonly in patients with a history of genetic abnormalities, radiation, or injury. More often than not, no specific cause is found.

Case An 80-year-old woman presents for a regular check up. She has been feeling fatigued more frequently during the last few months, sometimes wakes up with puffy eyes, and feels constipated once in a while. She is overweight and has a personal history of type 2 diabetes, hypertension, and high cholesterol. Thyroid function tests (TFTs) reveal a TSH level of 8.7U/mL and FT4 of 1.2ng/dL (reference normal values: TSH 0.4-4.2 U/mL; FT4 0.7-1.9 ng/dL). She is mentally alert but would like to feel less tired and be more active physically. Question What is the best treatment option for this patient? 100 ug (full dose)/day levothyroxine and regular TFTs. 50 ug (half dose)/day levothyroxine and regular TFTs. Daily full dose of levothyroxine for 3 months, followed by TFTs. Stepwise increase of levothyroxine daily dose (up to full dose) and TFTs. Thyroid function tests (TFTs) at 3 month intervals.

Correct answer: Thyroid function tests (TFTs) at 3 month intervals. Explanation The answer thyroid function tests (TFTs) at 3-month intervals is the correct answer. Based on her symptoms and lab tests, the patient could be diagnosed with subclinical hypothyroidism. However, she is an elderly patient for which current guidelines do not recommend routine thyroid hormone substitution, if they show normal FT4 levels and a TSH below 10 mU/L. Thyroid function tests at regular intervals are recommended to monitor the course of the serum TSH level. In patients over 75 years, thyroid hormone substitution is not considered beneficial, as studies have shown an association between high TSH and a lower mortality rate. The answers 100 ug (full dose)/day levothyroxine and regular TFTs and 50 ug (half dose)/day levothyroxine and regular TFTs are incorrect. Current recommendations for the treatment of hypothyroidism require thyroid hormone replacement therapy, but in this case the laboratory results are consistent with subclinical hypothyroidism, for which such treatment is not recommended. Daily full dose of levothyroxine for 3 months, followed by TFTs is incorrect. A trial of thyroid hormone substitution treatment for a few months can be considered in younger individuals with hypothyroid symptoms, but not in the elderly. Stepwise increase of levothyroxine daily dose (up to full dose) and TFTs is incorrect. Administration of the thyroid hormone replacement therapy can be started with a lower dose in some patients, but the lower dose is maintained during the treatment in elderly patients. In patients over 75 years, no treatment and TSH level monitoring is preferable.

Case A 28-year-old man presents with severe headaches. He states that they began a few weeks ago and that he has been taking over-the-counter medications with some relief. He reports that he has AIDS. You ask what the patient was doing prior to experiencing these headaches. He says that he was taking care of the neighbor's cat while they were on vacation. Concerned, you order a CT scan; it demonstrates multiple ring-enhanced lesions. Question What organism do you suspect? Histoplasma capsulatum Cryptococcus neoformans Toxoplasma gondii Cytomegalovirus (CMV) Herpes simplex virus

Correct answer: Toxoplasma gondii Explanation The clinical picture is suggestive of toxoplasmic encephalitis. The causative organism is Toxoplasma gondii. The definitive hosts are cats. In immunocompromised patients, imaging of the brain is warranted. The CT or MRI typically shows multiple ring-enhancing cerebral lesions. In immunocompromised patients, histoplasmosis is usually disseminated with primary lung and multiple organ system involvement. Also, the primary vector is bird droppings and bat exposure in the Ohio and Mississippi River valleys. Findings with cryptococcus include headache, abnormal mental status, and meningismus. Intracerebral mass lesions (cryptococcomas) are rarely seen. Neurologic CMV symptoms in the immunocompromised patient include polyradiculopathy, transverse myelitis, ventricular encephalitis, and focal encephalitis. There is no indication that CMV produces cerebral lesions. Herpes simplex encephalitis presents with nonspecific symptoms. A flu-like prodrome is followed by a headache, fever, behavioral and speech disturbances, and seizures. Cerebral lesions are not seen on CT.

Case A previously healthy 8-year-old African American boy presented with a 3-day history of worsening fatigue and generalized edema. You saw him about a week ago when he had symptoms of a cold, for which you advised only supportive therapy. He appears alert and cooperative. His vitals are normal (temperature 37°C, pulse 90/min, respiratory rate 20/min, and blood pressure 100/70 mm Hg). Physical examination reveals the presence of generalized edema, and the rest of the examination is within normal limits. His laboratory values are below: Test Finding Normal (for age) Units Urine protein dipstick 4+ 0 Urine protein 40 0-20 mg/dL Specific gravity 1030 1008-1020 Urine Protein/creatinine ratio 2 <0.2 Per gr creatinine Serum protein 2.9 5.9-8 g/dL Cholesterol 299 112-247 mg/dL Urea nitrogen 10 17-Jul mg/dL Creatinine 0.5 0.3-0.9 mg/dL The rest of his laboratory results are non-contributory. You plan to start therapy with prednisone. Question What parameter would be most important to follow and evaluate the effect of therapy in order to prevent chronic kidney disease? Edema Serum protein Blood pressure Urine protein Lipids

Correct answer: Urine protein Explanation Your patient most likely has nephrotic syndrome. The diagnostic criteria for the nephrotic syndrome are generalized edema, hypoproteinemia with disproportionately low albumin level (<3 grams/dL), urine protein to urine creatinine ratio in excess of 2 mg/dL in a first morning void or a 24-hour urine protein loss that exceeds 50 mg/kg or 40 mg/m2, and hypercholesterolemia (>200 mg/dL). He is normotensive, and the effect of therapy on proteinuria will be the best indicator of therapeutic efficacy. Serum protein will probably normalize when the patient stops losing proteins. Your patient is normotensive. If he develops hypertension, then normalization of blood pressure would be an additional important factor that should be considered in the decision about the therapy. Edema is important for the evaluation of a need for diuretics, but it is not a direct parameter to be considered for the progression of chronic kidney disease. Small studies with statins in nephrotic syndrome in children demonstrated their efficacy in lowering hyperlipidemia, but there were no changes in proteinuria, hypoalbuminemia, or progression of renal disease. Lipid abnormalities will resolve when the nephrotic syndrome is in remission.

A 64-year-old woman presents with urine leakage; it has been progressively worsening for the past 6-12 months. She also admits to having vaginal dryness, poor lubrication with intercourse, and subsequent dyspareunia. Urinary leakage primarily seems to occur with coughing, sneezing, or high-impact exercise. She denies dysuria, hematuria, urgency, and vasomotor symptoms. She tries to empty her bladder every 2-3 hours and has cut out all alcoholic and caffeinated beverages; however, symptoms still persist. The patient is a G6P6. She has no chronic medical conditions and no past surgeries. She wants to do everything she can to avoid surgery. She is in a monogamous relationship. She was last seen 1 year ago, with a normal well-woman exam and normal Pap smear. On physical exam, vitals are normal. The pelvic exam reveals dry, pale vaginal mucosa with a bulge on the anterior vaginal wall; it increases in size with a Valsalva maneuver. Urinalysis is normal. Question Although the provider can counsel the patient that surgery is the only definitive treatment for her condition, what intervention is the most likely to improve her urinary symptoms? Vaginal pessary Oral nitrofurantoin (Macrobid) Oral pentosan polysulfate sodium (Elmiron) Oral phenazopyridine (Pyridium or Azo) Oral tolterodine (Detrol)

Correct answer: Vaginal pessary Explanation This patient's urinary symptoms are secondary to a cystocele, which is a bulging of the bladder into the vagina. This patient's pelvic floor musculature has been weakened by delivery of 6 children and her symptoms are further aggravated by post-menopausal vaginal atrophy. Stress incontinence is one of the most common symptoms associated with having a cystocele. If she is a good candidate for surgery, the most effective treatment for this patient's cystocele is surgical repair. However, most short- and medium-term studies note improved symptoms in women treated with a pessary. After 2-4 months of pessary use, observational studies have reported resolution of prolapse symptoms (bulge, pressure, splinting) in 70-90% of women, and resolution of associated urinary symptoms (stress and urgency incontinence and voiding difficulty) in 40-50% of women. Topical estrogen cream may also help with the vaginal dryness and might provide some mild relief of urinary symptoms, but it does not address the root cause of the problem. Oral nitrofurantoin (Macrobid) is an antibiotic, which would be appropriate for treatment (or prophylaxis) of urinary tract infection (UTI). This patient does not present with typical UTI symptoms of dysuria, frequency, and urgency; in addition, she does not have an abnormal urinalysis. Oral pentosan polysulfate sodium (Elmiron) is a medication used for interstitial cystitis (IC), which is a chronic inflammatory disorder of the bladder. Classically, IC presents with nocturia, urgency, frequency, pain with a full bladder, and sometimes dyspareunia. IC is a diagnosis of exclusion. This patient has a physical exam finding of a cystocele and vaginal atrophy, which explains her symptoms. Pentosan polysulfate sodium would not be appropriate. Oral phenazopyridine (Pyridium or Azo) is a medication frequently used for relief of dysuria associated with UTI. This patient denies dysuria, so this medication would not be indicated. Oral tolterodine (Detrol) is an anticholinergic medication used for urge incontinence and overactive bladder (OAB) syndrome. This patient denies urge incontinence and OAB symptoms.

A 28-year-old woman presents with malaise. She is known to be HIV positive. Her CD4 count is unchanged at 350 cells/field, and her viral count is undetectable. She is afebrile and has a normal exam. She takes zidovudine, indinavir, potassium, hydrochlorothiazide, and glyburide. Workup shows: Serum bicarbonate 20 meq/L Serum sodium 140 meq/L Serum chloride 100 meq/L Serum lactate 6 mmol/L Serum potassium 4.2 meq/L Whole blood glucose 85 mg/dL HemoglobinA1C 6.2% Hemoglobin 13.5 g/dL( unchanged) Question What medication is likely causing her elevated lactate? Hydrochlorothiazide Glyburide Indinavir Zidovudine Potassium

Correct answer: Zidovudine Explanation Lactic acidosis is a known complication of zidovudine use. Zidovudine is a nucleoside analog reverse-transcriptase inhibitor (NRTI) used in the anti-retroviral treatment of AIDS and HIV. NRTI are thought to cause mitochondrial toxicity, leading to lactic acidosis. Lactic acidosis is a form of elevated anion gap metabolic acidosis. Patients with NRTI-induced lactic acidosis may range from having asymptomatic mild and chronically elevated lactate levels to life-threatening acute elevations (Berns) with associated cardiac and hemodynamic complications. Glyburide, potassium, and hydrochlorothiazide are not likely to cause lactic acidosis. Hydrochlorothiazide may increase serum bicarbonate and decrease serum potassium. Indinavir is a protease inhibitor used to treat HIV and AIDS. Renal side effects of indinavir use include crystalluria and nephrolithiasis, but not lactic acidosis.

A 36-year-old G1P1001 woman presents with a 4- to 5-week history of pain along her right lateral wrist; the pain worsens when she tries to grasp something. There was no trauma. She gave birth 6 weeks ago and carrying her baby is difficult secondary to pain. She is right-handed and has never had any issues like this before. On exam, there is tenderness and edema over the radial styloid. You then have the patient fully flex her thumb, adduct, and grasp it with that hand. You then place her hand in ulnar deviation, which reproduces the pain described above. Question Ico-delete Highlights What is the most likely diagnosis? Rheumatoid arthritis Ganglion cyst Scaphoid fracture de Quervain tenosynovitis Radial nerve entrapment

Correct answer: de Quervain tenosynovitis Explanation de Quervain Tenosynovitis is a tenosynovitis of the abductor pollicis longus and extensor pollicis brevis tendons at the radial styloid process. It is most commonly seen in women 35-55. It can happen when women carry children with their thumb overextended. On exam, there can be tenderness and swelling over the radial styloid. There is also a positive Finkelstein test, which is what is described in the scenario. Rheumatoid arthritis usually affects the small bones in the hands and is bilateral. Patients have morning stiffness. Ganglion cyst frequently presents with a lump in the wrist, which is inconsistent with the scenario. Scaphoid fracture is an incorrect response, as our patient did not have tenderness over the scaphoid bone. She also did not have any trauma. A scaphoid fracture most commonly occurs by falling on an outstretched hand. Radial nerve entrapment usually causes burning pain and paresthesia. A positive Tinel's sign would be seen.

A 20-year-old Caucasian man presents with a 1-month history of tingling sensations in his legs. He is on isoniazid treatment for pulmonary tuberculosis. On examination, you note a greasy yellow scale over his scalp and eyebrows. Laboratory investigations reveal a hemoglobin level of 9 g/dL. Question Ico-delete Highlights What is the most likely diagnosis? Vitamin A deficiency Pyridoxine deficiency Vitamin C deficiency Niacin deficiency Vitamin E deficiency

Pyridoxine deficiency Explanation In cases of pyridoxine or (vitamin B6 deficiency), patients can present with peripheral neuropathy, seborrheic dermatosis, glossitis, and cheilosis. Laboratory investigations reveal anemia with lymphopenia. Causes include malabsorption as well as medications such as isoniazid and penicillamine. Dietary sources of vitamin B6 include liver, legumes, whole grain cereals, and meats. In cases of vitamin A deficiency, patients can present with the inability to see well in dim light or night blindness. There may also be conjunctival and corneal xerosis, as well as pericorneal and corneal opacities, and Bitot's spots. Bitot's spots are a collection of keratin appearing as triangular foamy spots on the conjunctiva. The patient may also have xeroderma, hyperkeratotic skin lesions, and increased susceptibility to infections. Causes include inadequate dietary intake and malabsorption. Dietary sources of vitamin A include fish, liver, egg yolks, butter, cream, dark green leafy vegetables, and yellow fruits and vegetables. In cases of vitamin C (or ascorbic acid) deficiency, patients can present with bleeding tendencies (as a result of weakened capillaries) and impaired wound healing due to impaired formation of connective tissue. On examination, the gums may be swollen and friable; the teeth may be loose. There may also be multiple splinter hemorrhages on the nails and ecchymoses, especially over the lower limbs. Causes include inadequate dietary intake and certain conditions such as pregnancy and lactation that increase vitamin C requirements. Dietary sources of vitamin C include citrus fruits, such as oranges, lemons, and tangerines, as well as tomatoes and potatoes. Niacin deficiency causes pellagra, which is characterized by: Symmetrical dermatitis, usually on parts of the body exposed to sunlight Scarlet glossitis and stomatitis Diarrhea Mental aberrations, such as memory impairment, depression, and dementia. These may appear alone or in combination. Causes include inadequate dietary intake, especially in patients with corn-based diets or alcoholism. Dietary sources include legumes, yeast, meat, and enriched cereal products. Vitamin E deficiency may cause a hemolytic anemia in premature infants. Laboratory investigations reveal low plasma tocopherol levels, a low hemoglobin level, reticulocytosis, hyperbilirubinemia, and creatinuria. Causes of vitamin E deficiency in premature infants include a limited placental transfer of vitamin E and the resultant low levels at birth combined with its relative deficiency in the infant diet. Dietary sources for older children and adults include wheat germ, vegetable oils, egg yolk, and leafy vegetables.


Set pelajaran terkait

BIOL252 07: Chpt 10 appendicular skeleton

View Set

Microbiology Chapter 14: Innate Immune Response

View Set

Campbell Biology: Ninth Edition - Chapter 1: The Study of Life

View Set

SECTION 1: Events of Jesus' Life

View Set